0% found this document useful (0 votes)
525 views112 pages

Apush 2018 Pleasea

kjnpojpojop

Uploaded by

kadenpizza66
Copyright
© © All Rights Reserved
We take content rights seriously. If you suspect this is your content, claim it here.
Available Formats
Download as PDF, TXT or read online on Scribd
0% found this document useful (0 votes)
525 views112 pages

Apush 2018 Pleasea

kjnpojpojop

Uploaded by

kadenpizza66
Copyright
© © All Rights Reserved
We take content rights seriously. If you suspect this is your content, claim it here.
Available Formats
Download as PDF, TXT or read online on Scribd
You are on page 1/ 112

lOMoARcPSD|34158146

Apush 2018 - pleasea

United States, 1492-1865 (University of Texas at Austin)

Studocu is not sponsored or endorsed by any college or university


Downloaded by Loadster (kadenpizza66@gmail.com)
lOMoARcPSD|34158146

From the 2018 Administration

AP United States
History
Practice Exam

This exam may not be posted on Further distribution of these


school or personal websites, nor materials outside of the secure
electronically redistributed for College Board site disadvantages
any reason. This Released Exam is teachers who rely on uncirculated
provided by the College Board for questions for classroom testing.
AP Exam preparation. Teachers are Any additional distribution is in
permitted to download the materials violation of the College Board’s
and make copies to use with their copyright policies and may result
students in a classroom setting only. in the termination of Practice Exam
To maintain the security of this exam, access for your school as well as the
teachers should collect all materials removal of access to other online
after their administration and keep services such as the AP Teacher
them in a secure location. Community and Online Score Reports.

© 2018 The College Board. College Board, Advanced Placement Program, AP, AP Central, and the acorn logo
are registered trademarks of the College Board. Visit the College Board on the Web: www.collegeboard.org.
AP Central is the official online home for the AP Program: apcentral.collegeboard.org

Downloaded by Loadster (kadenpizza66@gmail.com)


lOMoARcPSD|34158146

Contents

Exam Instructions

Student Answer Sheet for the Multiple-Choice Section

Section I: Multiple-Choice and Short-Answer Questions

Section II: Free-Response Questions

Multiple-Choice Answer Key

Free-Response Scoring Guidelines

Scoring Worksheet

Question Descriptors and Performance Data

Note: This publication shows the page numbers that appeared in


the 2017−18 AP Exam Instructions book and in the actual exam.
This publication was not repaginated to begin with page 1.

© 2018 The College Board. College Board, Advanced Placement Program, AP, SAT and the acorn logo are
registered trademarks of the College Board. All other products and services may be trademarks of their
respective owners. Permission to use copyrighted College Board materials may be requested online at:
www.collegeboard.org/request-form.

Downloaded by Loadster (kadenpizza66@gmail.com)


lOMoARcPSD|34158146

Exam Instructions

The following contains instructions taken from


the 2017−18 AP Exam Instructions book.

Downloaded by Loadster (kadenpizza66@gmail.com)


lOMoARcPSD|34158146

AP United States
History Exam
Regularly Scheduled Exam Date: Friday morning, May 11, 2018
Late-Testing Exam Date: Thursday morning, May 24, 2018

Section I Total Time: 1 hour and 35 minutes Part A: Multiple-Choice Time: 55 minutes
Percent of Total Score: 60% Questions Writing Instrument:
Number of Questions: 55 Pencil required
(The number of questions
may vary slightly depending
on the form of the exam.)

Part B: Short-Answer Time: 40 minutes


Questions Writing Instrument: Pen
Number of Questions: 3 with black or dark blue ink
Questions 1 and 2:
Mandatory
Question 3 or 4:
Choose one question

Section II Total Time: 1 hour and 40 minutes Document-Based Question (DBQ)


Number of Questions: 1 Document- (15-minute reading period; 45-minute writing period)
based question and 1 long essay
Percent of Total Score: 40% Long Essay Question
Writing Instrument: Pen with (40-minute writing period)
black or dark blue ink

Before Distributing Exams: Check that the title on all exam covers is United States History. If there are any exam
booklets with a different title, contact the AP coordinator immediately.

What Proctors Need to Bring to This Exam


Exam packets Container for students’ electronic devices (if needed)
Answer sheets Extra No. 2 pencils with erasers
Short-Answer Response Booklet Extra pens with black or dark blue ink
Return Envelope(s) Lined paper
AP Student Packs Stapler
2017-18 AP Coordinator’s Manual Watch
This book—2017-18 AP Exam Instructions Signs for the door to the testing room
AP Exam Seating Chart template – “Exam in Progress”
School Code and Homeschool/Self-Study Codes – “Cell phones are prohibited during the test
Pencil sharpener administration, including breaks”

Downloaded by Loadster (kadenpizza66@gmail.com)


lOMoARcPSD|34158146

2017–18 AP Exam Instructions

SEATING POLICY FOR AP UNITED STATES HISTORY EXAM


Exams Administered at Exams Administered at
Schools in the United States, Schools Outside the United
Canada, Puerto Rico, and States, Canada, Puerto Rico,
Testing Window the U.S. Virgin Islands and the U.S. Virgin Islands
Regularly Students must be seated
Scheduled Exams no less than 4 feet apart. Students must be seated
Late-Testing Students must be seated no less than 5 feet apart.
Exams no less than 5 feet apart.

SECTION I: Multiple Choice and Short Answer


› Do not begin the exam instructions below until you have completed the
appropriate General Instructions for your group.

Make sure you begin the exam at the designated time. Remember, you must complete
a seating chart for this exam. See pages 303–304 for a seating chart template and
instructions. See the 2017-18 AP Coordinator’s Manual for exam seating requirements
(pages 55–58).

If you are giving the regularly scheduled exam, say:


It is Friday morning, May 11, and you will be taking the AP United States
History Exam.

If you are giving the alternate exam for late testing, say:
It is Thursday morning, May 24, and you will be taking the AP United States
History Exam.

Look at your exam packet and confirm that the exam title is “AP United States
History.” Raise your hand if your exam packet contains any title other than
“AP United States History,” and I will help you.

Once you confirm that all students have the correct exams, say:
In a moment, you will open the exam packet. By opening this packet, you
agree to all of the AP Program’s policies and procedures outlined in the
2017-18 Bulletin for AP Students and Parents.
You may now remove the shrinkwrap from your exam packet and take out the
two Section I booklets. One is labeled Section I: Multiple Choice and Short
Answer, and the other is labeled Section I, Part B: Short-Answer Response
Booklet. Do not open the short-answer response booklet or the shrinkwrapped
Section II materials.
Place the short-answer response booklet and the seals on top of the
shrinkwrapped Section II exam materials, and set these aside on your desk.
You should now refer only to the Section I: Multiple Choice and Short Answer
booklet and your multiple-choice answer sheet. . . .
Carefully remove the AP Exam label found near the top left of your Multiple
Choice and Short Answer booklet cover. Place the label on page 1 of your
multiple-choice answer sheet on the light blue box near the top right corner
that reads “AP Exam Label.”

278 AP United States History Exam


Downloaded by Loadster (kadenpizza66@gmail.com)
lOMoARcPSD|34158146

2017–18 AP Exam Instructions

If students accidentally place the exam label in the space for the number label or vice

AP United States History Exam


versa, advise them to leave the labels in place. They should not try to remove the label;
their exam can still be processed correctly.

Listen carefully to all my instructions. I will give you time to complete each
step. Please look up after completing each step. Raise your hand if you have
any questions.
Give students enough time to complete each step. Don’t move on until all students are ready.

Read the statements on the front cover of the exam booklet. . . .


Sign your name and write today’s date. . . .
Now print your full legal name where indicated. . . .
Now turn to the back cover of your exam booklet and read it completely. . . .
Are there any questions? . . .
You will begin with the multiple-choice part of the exam. You may never discuss
the multiple-choice exam content at any time in any form with anyone, including
your teacher and other students. If you disclose the multiple-choice exam
content through any means, your AP Exam score will be canceled.
Put all pens aside. Does everyone have a pencil? . . .
Provide pencils as necessary.

Then say:
For U.S. History, you will use only the circles marked A–D on the answer sheet.
You must complete the answer sheet using a No. 2 pencil only. Mark one
response per question. Completely fill in the circles. If you need to erase, do
so completely. No credit will be given for anything written in the exam booklet.
Scratch paper is not allowed, but you may use the margins or any blank space in
the exam booklet for scratch work. Are there any questions? . . .
You have 55 minutes for the multiple-choice questions. If you finish before time is
called, you may check your work, but do not go on to Part B until I tell you to do so.
Your answer sheets will be collected at the end of the 55 minutes. Open your exam
booklet and begin.

Note Start Time . Note Stop Time .

Proctors should circulate throughout the testing room to check that students are marking
their answers in pencil on their multiple-choice answer sheets and that they are not
looking ahead to Part B or at the other exam booklets. Proctors should stop any students
who begin reading the Part B short-answer questions or begin writing in the Part B:
Short-Answer Response Booklet (this booklet should remain on top of the Section II
shrinkwrapped packet). These students should be redirected back to the multiple-choice
questions. If students complete the multiple-choice questions early they may recheck their
work on Part A or sit quietly until you give the next instruction.

After 45 minutes, say:


There are 10 minutes remaining.

After 10 minutes, say:


Stop working. Close your exam booklet and put your multiple-choice answer
sheet on your desk, faceup. Make sure you have your AP number label and an
AP Exam label on page 1 of your answer sheet.
Sit quietly while I collect your answer sheets. Do not reopen the exam booklet
until you are told to do so.

AP United States History Exam 279


Downloaded by Loadster (kadenpizza66@gmail.com)
lOMoARcPSD|34158146

2017–18 AP Exam Instructions

Collect a multiple-choice answer sheet from each student. Check that each answer sheet
has an AP number label and an AP Exam label.

After all multiple-choice answer sheets have been collected, say:


Now put all pencils aside. Does everyone have a pen with black or dark
blue ink? . . .
Get your Student Pack from under your chair, and get the short-answer
response booklet that you set aside earlier. Leave the seals and the
shrinkwrapped Section II packet to the side. . . .
You should now have in front of you:
ƒ your Student Pack,
ƒ the Section I: Multiple Choice and Short Answer booklet, and
ƒ the short-answer response booklet.
For this part of the exam you need to answer 3 questions—Questions 1 and 2
are mandatory, then you must choose between answering Question 3 or
Question 4.
Look at the cover of the short-answer response booklet. Take an AP number
label from your Student Pack and place it in the box at the top of the page. Write
your AP number if you don’t have any labels. . . .
Now complete items 1 through 4 in the Important Identification Information. . . .
Now read the bulleted instructions under “As you complete this short-answer
response booklet.”. . .
This booklet has lined pages that are labeled with watermarks “Q1,” ”Q2,” and
“Q3 or Q4” that correspond to the short-answer questions you will answer. Are
there any questions? . . .
Place your Student Pack under your chair. . . .
The short-answer questions are in Part B of the Section I: Multiple Choice and
Short Answer booklet. Read the questions carefully. Your responses should
be brief; they should not be essays. Longer responses will not necessarily
receive higher scores than shorter ones that accomplish all the tasks set by the
question.
Remember—Questions 1 and 2 are mandatory, and you should choose to
answer either Question 3 or Question 4.
You must write your response to each question on the correct designated page.
You have 40 minutes for this part. Are there any questions before we begin? . . .
Turn to Part B in the exam booklet and make sure you have the short-answer
response booklet turned to page Q1. Begin working.

Note Start Time . Note Stop Time .

Check that students are writing their answers in their short-answer response booklet on
the lined pages designated for the questions. The booklet is designed to provide sufficient
space for responses. However, if a student requires additional space to complete responses,
provide lined paper. Instruct those students to write only their AP number, the exam section
(Section I, Part B), and the number of the question they are working on, at the top of each
extra sheet of paper they use. The shrinkwrapped Section II materials should still be on the
side and unopened.

280 AP United States History Exam


Downloaded by Loadster (kadenpizza66@gmail.com)
lOMoARcPSD|34158146

2017–18 AP Exam Instructions

After 30 minutes, say:

AP United States History Exam


There are 10 minutes remaining.

After 10 minutes, say:


Stop working and close both booklets. Check that you filled in the circle
indicating Question 3 or Question 4 on the last page of the short-answer
response booklet.
If any students used extra paper for a question in the short-answer part, have those
students staple the extra sheet(s) to the page corresponding to that specific question in their
short-answer response booklets. Complete an Incident Report after the exam (see page 67 of
the 2017-18 AP Coordinator’s Manual for complete details).

Then say:
I will now collect the short-answer response booklet.
Collect a short-answer response booklet from each student. Check for the following:
ƒ Short-answer response booklet cover: The student placed an AP number label on the
shaded box and completed the identification information.
ƒ Short-answer response booklet page “Q3 or Q4”: The student filled in the circle
indicating whether they answered question 3 or question 4.
Keep these short-answer response booklets separate from the multiple-choice answer
sheets. Before leaving the testing room, you will need to insert all of the short-answer
response booklets into the yellow short-answer response booklet return envelope(s).
Reminder, if any students used extra sheets of paper, those short-answer response booklets
must be returned in the Incident Report return envelope.

Now you must seal your exam booklet using the white seals you set aside
earlier. Remove the white seals and press one on each area of your Section I
booklet cover marked “PLACE SEAL HERE.” Fold each seal over the back cover.
When you have finished, place the booklet on your desk, faceup. I will now
collect your exam booklet. . . .
Collect the Section I: Multiple Choice and Short Answer booklet from each student. Check
that all booklets are sealed and that students have signed the front cover of the booklets.

There is a 10-minute break between Sections I and II.

When all Section I materials have been collected and accounted for and you are
ready for the break, say:
Please listen carefully to these instructions before we take a 10-minute break.
All items you placed under your chair at the beginning of this exam must stay
there, and you are not permitted to open or access them in any way. Leave your
shrinkwrapped Section II packet on your desk during the break. You are not
allowed to consult teachers, other students, notes, or textbooks during the
break. You may not make phone calls, send text messages, check email, use
a social networking site, or access any electronic or communication device.
Remember, you may never discuss the multiple-choice exam content with
anyone, and if you disclose the content through any means, your AP Exam score
will be canceled.

AP United States History Exam 281


Downloaded by Loadster (kadenpizza66@gmail.com)
lOMoARcPSD|34158146

2017–18 AP Exam Instructions

If you are giving the regularly scheduled exam, say:


You may not discuss or share any of the short-answer exam content with anyone
unless it is released on the College Board website in about two days.

If you are giving the alternate exam for late testing, say:
In addition, none of the short-answer content in this exam may ever be
discussed or shared in any way at any time. If you disclose the short-answer
exam content through any means, your AP Exam score will be canceled.

Are there any questions? . . .

You may begin your break. Testing will resume at .

SECTION II: Free Response


Section II (free response) of this exam begins with a mandatory document-based question
(Question 1) and then gives students a choice of three essay topics (Question 2, Question 3,
or Question 4). A sheet of essay-choice labels is included with the Section II free-response
booklets. At the conclusion of the free-response section, you will instruct students to apply
the appropriate label where indicated on the front cover of their Section II exam booklet,
identifying their essay choice.

After the break, say:


May I have everyone’s attention? Place your Student Pack on your desk. . . .
Remove the shrinkwrap from the Section II packet, but do not open either the
Section II exam booklet or the orange Section II: Free Response, Questions and
Documents booklet until you are told to do so. Put the essay-choice labels aside.
You will need them at the end of Section II. . . .
Read the bulleted statements on the front cover of the exam booklet. Look up
when you have finished. . . .
Now take an AP number label from your Student Pack and place it on the
shaded box. If you don’t have any AP number labels, write your AP number in
the box. . . .
Read the last statement. . . .
Using your pen, print the first, middle, and last initials of your legal name in the
boxes and print today’s date where indicated. This constitutes your signature and
your agreement to the statements on the front cover. . . .
Turn to the back cover and, using your pen, complete the “Important
Identification Information.”. . .
If this is your last AP Exam, you may keep your Student Pack. Place it under your
chair for now. Otherwise I will collect all Student Packs.
Read the information on the back cover of the Section II: Free Response
booklet. Do not open the booklet until you are told to do so. . . .
Collect the Student Packs.

Then say:
Now, read the information on the front cover of the orange booklet. . . .

282 AP United States History Exam


Downloaded by Loadster (kadenpizza66@gmail.com)
lOMoARcPSD|34158146

2017–18 AP Exam Instructions

Section II is the free-response portion of the exam. You have 1 hour and

AP United States History Exam


40 minutes. This includes a 15-minute reading period. The reading period
is designed to provide you with time to develop thoughtful, well-organized
responses. During the reading period you are advised to read Question 1 and
plan your answer. If you have time, you may also read Questions 2, 3, and 4.
You may begin writing your responses before the reading period is over. You
may make notes in the orange booklet, but your responses must be written
in the free-response booklet using your pen. At the top of each page in the
free-response booklet, circle the number of the question you are answering.
You must answer Question 1, the document-based question, and you must
choose one of the three long essay questions to answer.
The suggested writing time for Question 1 is 45 minutes. After 45 minutes,
you will be advised to move on to the next question. Questions 2, 3, and 4 are
weighted equally; you won’t get extra credit for answering the question that may
seem more difficult. Choose the question for which you are best prepared.
If you need more paper to complete your responses, raise your hand. At the top
of each extra sheet of paper you use, write only:
ƒ your AP number,
ƒ Section II, and
ƒ the question number you are working on.
You may now open the orange booklet and exam booklet and begin the
15-minute reading period.

Note Start Time . Note Stop Time .

After 15 minutes, say:


The reading period is over. You have 1 hour and 25 minutes remaining to
complete Section II.

Note Start Time . Note Stop Time .

Check that students are using pens and that they are writing their answers in their exam
booklets and not in the orange booklets.

After 45 minutes, say:


You are advised to move on to the next question. Answer only Question 2, 3, or 4.
Do not answer more than one long essay question.

After 30 minutes, say:


There are 10 minutes remaining.

After 10 minutes, say:


Stop working and close both booklets. Look at the bottom front cover of
your free-response booklet. Using the essay-choice labels that you set
aside earlier, select the label that matches the question you chose to answer
(Question 2, Question 3, or Question 4) and apply it to the bottom front cover
of your free-response booklet, where indicated. For example, if you answered
Question 2 apply the label “2.” Look up when you have finished. . . .
Now put both booklets on your desk, faceup. Do not place your Section II
free-response booklet inside your orange booklet or vice versa. Put your unused
labels next to the booklets. . . .

AP United States History Exam 283


Downloaded by Loadster (kadenpizza66@gmail.com)
lOMoARcPSD|34158146

2017–18 AP Exam Instructions

If any students used extra paper for a question in the free-response section, have those
students staple the extra sheet(s) to the first page corresponding to that question in their
free-response booklets. Complete an Incident Report after the exam (see page 67 of the
2017-18 AP Coordinator’s Manual for complete details).

Then say:
Remain in your seat, without talking, while the exam materials are collected. . . .
Collect a free-response booklet, an orange booklet, and the unused labels from each
student. Check for the following:

ƒ Free-response booklet front cover: The student placed an AP number label and an
essay-choice label in the appropriate boxes and printed their initials and today’s date.
ƒ Free-response booklet back cover: The student completed the “Important Identification
Information” area.
ƒ The student wrote answers in the free-response booklet and not in the orange booklet.
The orange booklets must be returned with the rest of your exam materials. Keep the
orange booklets separate from the free-response booklets. Do not place the free-response
booklets inside the orange booklets or vice versa.

You may discard the essay-choice labels after you have checked that all students have
applied one label to each free-response booklet. When all exam materials have been
collected and accounted for, return to students any electronic devices you may have
collected before the start of the exam.

If you are giving the regularly scheduled exam, say:


You may not discuss or share the free-response exam content with anyone
unless it is released on the College Board website in about two days. Your AP
Exam score results will be available online in July.

If you are giving the alternate exam for late testing, say:
None of the content in this exam may ever be discussed or shared in any way at
any time. Your AP Exam score results will be available online in July.

If any students completed the AP number card at the beginning of this exam, say:
Please remember to take your AP number card with you. You will need the
information on this card to view your scores and order AP score reporting
services online.

Then say:
You are now dismissed.

After-Exam Tasks
Before you leave the testing room, be sure to insert all of the Section I, Part B: Short-Answer
Response Booklets inside the yellow short-answer response booklet return envelope(s) with
the folds of the short-answer response booklets facing the same direction. Insert no more
than 25 short-answer response booklets in a single envelope. Do not include multiple-choice
answer sheets inside the short-answer response booklet return envelope(s). If you use more
than one envelope, number the envelopes consecutively (1 of 3, 2 of 3, 3 of 3, etc.).

Do not tape or seal this envelope. Follow the directions on pages 75–76 of the 2017-18
AP Coordinator’s Manual for packing the yellow short-answer response booklet return
envelope(s) in your return shipment to the AP Program.

284 AP United States History Exam


Downloaded by Loadster (kadenpizza66@gmail.com)
lOMoARcPSD|34158146

2017–18 AP Exam Instructions

Be sure to give the completed seating chart to the AP coordinator. Schools must retain

AP United States History Exam


seating charts for at least six months (unless the state or district requires that they be
retained for a longer period of time). Schools should not return any seating charts in their
exam shipments unless they are required as part of an Incident Report.

NOTE: If you administered exams to students with accommodations, review the 2017-18 AP
Coordinator’s Manual and the 2017-18 AP SSD Guidelines for information about completing
the NAR form, and returning these exams.

The exam proctor should complete the following tasks if asked to do so by the
AP coordinator. Otherwise, the AP coordinator must complete these tasks:

ƒ Complete an Incident Report for any students who used extra paper for the free-response
section. (Incident Report forms are provided in the coordinator packets sent with
the exam shipments.) These forms must be completed with a No. 2 pencil. It is
best to complete a single Incident Report for multiple students per exam subject, per
administration (regular or late testing), as long as all required information is provided.
Include all exam booklets with extra sheets of paper in an Incident Report return
envelope (see page 67 of the 2017-18 AP Coordinator’s Manual for complete details).
ƒ Return all exam materials to secure storage until they are shipped back to the
AP Program. (See page 26 of the 2017-18 AP Coordinator’s Manual for more information
about secure storage.) Before storing materials, check the “School Use Only” section on
page 1 of the answer sheet and:
Š Fill in the appropriate section number circle in order to access a separate
AP Instructional Planning Report (for regularly scheduled exams only) or subject
score roster at the class section or teacher level. See “Post-Exam Activities” in the
2017-18 AP Coordinator’s Manual.
Š Check your list of students who are eligible for fee reductions and fill in the
appropriate circle on their registration answer sheets.

AP United States History Exam 285


Downloaded by Loadster (kadenpizza66@gmail.com)
lOMoARcPSD|34158146

Student Answer Sheet for


the Multiple-Choice Section

Use this section to capture student responses. (Note that the following
answer sheet is a sample, and may differ from one used in an actual exam.)

Downloaded by Loadster (kadenpizza66@gmail.com)


lOMoARcPSD|34158146

AP Exam Label PAGE 1


AP Number Label
(from Student Pack) (from Section I Booklet)
Answer Sheet 2018
AP FPO X123456789 X
COMPLETE THIS AREA AT EVERY EXAM. USE NO. 2 PENCIL ONLY E. EXAM G. ONLINE
D. EXAM DATE F. MULTIPLE-CHOICE BOOKLET
I am aware of and agree to follow the policies and procedures in the 2017-18 Bulletin for AP Students and Parents to C. YOUR AP NUMBER START PROVIDER
Month Day TIME SERIAL NUMBER CODE
maintain the security of the exam and the validity of my AP score. I understand and accept that my exam score may
be canceled if I do not follow these policies and procedures, if I disregard any exam day instructions, or if my exam is AM PM S
administered improperly. I certify that I am the person whose information appears on this answer sheet.
Sign your legal name as it will appear on your college applications. Date
0 0 0 0 0 0 0 0 0 0 0 0 6 12 0 0 0 0 0 0 0 0 0 0 0 0 0
A. SIGNATURE
1 1 1 1 1 1 1 1 1 1 1 1 7 1 1 1 1 1 1 1 1 1 1 1 1 1 1

2 2 2 2 2 2 2 2 2 2 2 8 2 2 2 2 2 2 2 2 2 2 2 2 2 2
B. LEGAL NAME Omit apostrophes, Jr., II. 3 3 3 3 3 3 3 3 3 3 3 9 3 3 3 3 3 3 3 3 3 3 3 3 3 3
Legal Last Name — First 15 Letters Legal First Name — First 12 Letters MI 4 4 4 4 4 4 4 4 4 4 10 4 4 4 4 4 4 4 4 4 4 4 4 4 4

5 5 5 5 5 5 5 5 5 5 11 5 5 5 5 5 5 5 5 5 5 5 5 5 5
A A A A A A A A A A A A A A A A A A A A A A A A A A A A 6 6 6 6 6 6 6 6 6 6 6 6 6 6 6 6 6 6 6 6 6 6 6
B B B B B B B B B B B B B B B B B B B B B B B B B B B B 7 7 7 7 7 7 7 7 7 7 7 7 7 7 7 7 7 7 7 7 7 7 7
C C C C C C C C C C C C C C C C C C C C C C C C C C C C 8 8 8 8 8 8 8 8 8 8 8 8 8 8 8 8 8 8 8 8 8 8 8
D D D D D D D D D D D D D D D D D D D D D D D D D D D D 9 9 9 9 9 9 9 9 9 9 9 9 9 9 9 9 9 9 9 9 9 9 9
E E E E E E E E E E E E E E E E E E E E E E E E E E E E
COMPLETE THIS AREA ONLY ONCE. K. DATE OF BIRTH
F F F F F F F F F F F F F F F F F F F F F F F F F F F F I. AREA CODE AND
G G G G G G G G G G G G G G G G G G G G G G G G G G G G PHONE NUMBER J. SCHOOL YOU ATTEND Month Day Year
SCHOOL CODE School Name
H H H H H H H H H H H H H H H H H H H H H H H H H H H H Jan
I I I I I I I I I I I I I I I I I I I I I I I I I I I I 0 0 0 0 0 0 0 0 0 0 Feb 0 0 0 0
J J J J J J J J J J J J J J J J J J J J J J J J J J J J 1 1 1 1 1 1 1 1 1 1 0 0 0 0 0 0 Mar 1 1 1 1
K K K K K K K K K K K K K K K K K K K K K K K K K K K K 2 2 2 2 2 2 2 2 2 2 1 1 1 1 1 1 Apr 2 2 2 2
L L L L L L L L L L L L L L L L L L L L L L L L L L L L 3 3 3 3 3 3 3 3 3 3 2 2 2 2 2 2 May 3 3 3 3
M M M M M M M M M M M M M M M M M M M M M M M M M M M M 4 4 4 4 4 4 4 4 4 4 3 3 3 3 3 3 City Jun 4 4 4
N N N N N N N N N N N N N N N N N N N N N N N N N N N N 5 5 5 5 5 5 5 5 5 5 4 4 4 4 4 4 Jul 5 5 5
O O O O O O O O O O O O O O O O O O O O O O O O O O O O 6 6 6 6 6 6 6 6 6 6 5 5 5 5 5 5 State Aug 6 6 6
P P P P P P P P P P P P P P P P P P P P P P P P P P P P 7 7 7 7 7 7 7 7 7 7 6 6 6 6 6 6 Sep 7 7 7
Q Q Q Q Q Q Q Q Q Q Q Q Q Q Q Q Q Q Q Q Q Q Q Q Q Q Q Q 8 8 8 8 8 8 8 8 8 8 7 7 7 7 7 7 Oct 8 8 8
121586-00657 • TF11017E5500 • Printed in U.S.A.

Country
R R R R R R R R R R R R R R R R R R R R R R R R R R R R 9 9 9 9 9 9 9 9 9 9 8 8 8 8 8 8 Nov 9 9 9
S S S S S S S S S S S S S S S S S S S S S S S S S S S S INTERNATIONAL PHONE 9 9 9 9 9 9 Dec
T T T T T T T T T T T T T T T T T T T T T T T T T T T T
M. COLLEGE TO RECEIVE YOUR
U U U U U U U U U U U U U U U U U U U U U U U U U U U U N. CURRENT
L. SOCIAL SECURITY AP SCORE REPORT
GRADE LEVEL
V V V V V V V V V V V V V V V V V V V V V V V V V V V V NUMBER (Optional) COLLEGE CODE Using the college code listed
in the AP Student Pack,
indicate the ONE college Not yet in
W W W W W W W W W W W W W W W W W W W W W W W W W W W W 11th
that you want to receive 9th grade
X X X X X X X X X X X X X X X X X X X X X X X X X X X X 0 0 0 0 0 0 0 0 0 0 0 0 0 your AP score report. 9th 12th
Y Y Y Y Y Y Y Y Y Y Y Y Y Y Y Y Y Y Y Y Y Y Y Y Y Y Y Y 1 1 1 1 1 1 1 1 1 1 1 1 1 College Name 10th No longer in
high school
Z Z Z Z Z Z Z Z Z Z Z Z Z Z Z Z Z Z Z Z Z Z Z Z Z Z Z Z 2 2 2 2 2 2 2 2 2 2 2 2 2

– – – – – – – – – – – – – – – – – – – – – – – 3 3 3 3 3 3 3 3 3 3 3 3 3 O. STUDENT SEARCH SERVICE®


4 4 4 4 4 4 4 4 4 4 4 4 4 Colleges and scholarship programs may
Exam Name: Form: Form Code: request your information to inform you of
H. AP EXAM I AM City
TAKING USING THIS 5 5 5 5 5 5 5 5 5 5 5 5 5 educational opportunities and financial aid.
Q4546/1-4

ANSWER SHEET 6 6 6 6 6 6 6 6 6 6 6 6 6 Would you like us to supply your information?


State
7 7 7 7 7 7 7 7 7 7 7 7 7 Yes No
SCHOOL USE ONLY
801356 Section Number Fee Reduction Granted 8 8 8 8 8 8 8 8 8 8 8 8 8 Country
If you don’t answer and previously chose to
participate in this service, we will continue
1 2 3 4 5 6 7 8 9 1 Option 1 2 Option 2 9 9 9 9 9 9 9 9 9 9 9 9 9 providing your information.

Downloaded by Loadster (kadenpizza66@gmail.com)


lOMoARcPSD|34158146

PAGE 2
COMPLETE THIS AREA AT EACH EXAM (IF APPLICABLE).

P. SURVEY QUESTIONS — Answer the survey questions in the AP Student Pack. Do not put responses to exam questions in this section.

1 A B C D E F G H I 4 A B C D E F G H I 7 A B C D E F G H I

2 A B C D E F G H I 5 A B C D E F G H I 8 A B C D E F G H I

3 A B C D E F G H I 6 A B C D E F G H I 9 A B C D E F G H I

Q. LANGUAGE — Do not complete this section unless instructed to do so.

If this answer sheet is for the French Language and Culture, German Language and Culture, Italian Language and Culture, Spanish Language
and Culture, or Spanish Literature and Culture Exam, please answer the following questions. Your responses will not affect your score.
1. Have you lived or studied for one month or more in a country where the language of the 2. Do you regularly speak or hear the language at home?
exam you are now taking is spoken?

Yes No Yes No

QUESTIONS 1–75

Indicate your answers to the exam questions in this section (pages 2 and 3). Mark only one response per question
for Questions 1 through 120. If a question has only four answer options, do not mark option E. Answers written in
the multiple-choice booklet will not be scored.

A B C D You must use a No. 2 pencil and marks must be complete. Do not use a mechanical pencil. It
EXAMPLES OF
COMPLETE MARK is very important that you fill in the entire circle darkly and completely. If you change your response,
INCOMPLETE MARKS A B C D erase as completely as possible. Incomplete marks or erasures may affect your score.

1 A B C D E 26 A B C D E 51 A B C D E
2 A B C D E 27 A B C D E 52 A B C D E
3 A B C D E 28 A B C D E 53 A B C D E
4 A B C D E 29 A B C D E 54 A B C D E
5 A B C D E 30 A B C D E 55 A B C D E
6 A B C D E 31 A B C D E 56 A B C D E
7 A B C D E 32 A B C D E 57 A B C D E
8 A B C D E 33 A B C D E 58 A B C D E
9 A B C D E 34 A B C D E 59 A B C D E
10 A B C D E 35 A B C D E 60 A B C D E
11 A B C D E 36 A B C D E 61 A B C D E
12 A B C D E 37 A B C D E 62 A B C D E
13 A B C D E 38 A B C D E 63 A B C D E
14 A B C D E 39 A B C D E 64 A B C D E
15 A B C D E 40 A B C D E 65 A B C D E
16 A B C D E 41 A B C D E 66 A B C D E
17 A B C D E 42 A B C D E 67 A B C D E
18 A B C D E 43 A B C D E 68 A B C D E
19 A B C D E 44 A B C D E 69 A B C D E
20 A B C D E 45 A B C D E 70 A B C D E
21 A B C D E 46 A B C D E 71 A B C D E
22 A B C D E 47 A B C D E 72 A B C D E
23 A B C D E 48 A B C D E 73 A B C D E
24 A B C D E 49 A B C D E 74 A B C D E
25 A B C D E 50 A B C D E 75 A B C D E

FOR OFFICIAL USE ONLY


R W O R W O
0 1 2 3 4 5 6 7 8 9 SELECTED MEDIA EXAMS OTHER EXAMS
Exam
0 1 2 3 4 5 6 7 8 9
PT02 TOTAL
0 1 2 3 4 5 6 7 8 9
Exam PT03 Subscore (if applicable)
0 1 2 3 4 5 6 7 8 9
PT04 Subscore (if applicable)

DO NOT WRITE IN THIS AREA

Downloaded by Loadster (kadenpizza66@gmail.com)


lOMoARcPSD|34158146

PAGE 3
QUESTIONS 76–120

Be sure each mark is dark and completely fills the circle. If a question has only four answer options, do not mark option E.

76 A B C D E 91 A B C D E 106 A B C D E
77 A B C D E 92 A B C D E 107 A B C D E
78 A B C D E 93 A B C D E 108 A B C D E
79 A B C D E 94 A B C D E 109 A B C D E
80 A B C D E 95 A B C D E 110 A B C D E
81 A B C D E 96 A B C D E 111 A B C D E
82 A B C D E 97 A B C D E 112 A B C D E
83 A B C D E 98 A B C D E 113 A B C D E
84 A B C D E 99 A B C D E 114 A B C D E
85 A B C D E 100 A B C D E 115 A B C D E
86 A B C D E 101 A B C D E 116 A B C D E
87 A B C D E 102 A B C D E 117 A B C D E
88 A B C D E 103 A B C D E 118 A B C D E
89 A B C D E 104 A B C D E 119 A B C D E
90 A B C D E 105 A B C D E 120 A B C D E

QUESTIONS 121–126

For Students Taking AP Biology


Write your answer in the boxes at the top of the griddable area and fill in the corresponding circles.
Mark only one circle in any column. You will receive credit only if the circles are filled in correctly.

121 122 123 124 125 126

/ / / / / / / / / / / / / / / / / /

– . . . . . – . . . . . – . . . . . – . . . . . – . . . . . – . . . . .

0 0 0 0 0 0 0 0 0 0 0 0 0 0 0 0 0 0 0 0 0 0 0 0

1 1 1 1 1 1 1 1 1 1 1 1 1 1 1 1 1 1 1 1 1 1 1 1 1 1 1 1 1 1

2 2 2 2 2 2 2 2 2 2 2 2 2 2 2 2 2 2 2 2 2 2 2 2 2 2 2 2 2 2

3 3 3 3 3 3 3 3 3 3 3 3 3 3 3 3 3 3 3 3 3 3 3 3 3 3 3 3 3 3

4 4 4 4 4 4 4 4 4 4 4 4 4 4 4 4 4 4 4 4 4 4 4 4 4 4 4 4 4 4

5 5 5 5 5 5 5 5 5 5 5 5 5 5 5 5 5 5 5 5 5 5 5 5 5 5 5 5 5 5

6 6 6 6 6 6 6 6 6 6 6 6 6 6 6 6 6 6 6 6 6 6 6 6 6 6 6 6 6 6

7 7 7 7 7 7 7 7 7 7 7 7 7 7 7 7 7 7 7 7 7 7 7 7 7 7 7 7 7 7

8 8 8 8 8 8 8 8 8 8 8 8 8 8 8 8 8 8 8 8 8 8 8 8 8 8 8 8 8 8

9 9 9 9 9 9 9 9 9 9 9 9 9 9 9 9 9 9 9 9 9 9 9 9 9 9 9 9 9 9

QUESTIONS 131–142

For Students Taking AP Computer Science Principles, AP Physics 1, or AP Physics 2


Mark two responses per question. You will receive credit only if both correct responses are selected.

131 A B C D 135 A B C D 139 A B C D


132 A B C D 136 A B C D 140 A B C D
133 A B C D 137 A B C D 141 A B C D
134 A B C D 138 A B C D 142 A B C D

© 2017 The College Board. College Board, AP, Student Search Service and the acorn logo are registered trademarks of the College Board.

DO NOT WRITE IN THIS AREA

Downloaded by Loadster (kadenpizza66@gmail.com)


lOMoARcPSD|34158146

COMPLETE THIS AREA ONLY ONCE. PAGE 4


R. YOUR MAILING ADDRESS Use the address abbreviations from your AP Student Pack. Fill in only one circle per column. Indicate a space in your address by leaving a blank box; do not grid that column. V. SEX
COUNTRY
STREET ADDRESS (include street number, street name, apartment number, etc.) CITY ZIP OR POSTAL CODE CODE Female Male

W. WHICH LANGUAGE DO YOU


A A A A A A A A A A A A A A A A A A A A A A A A A A A A A A A A A A A A A A A A A A A A A A A A 0 0 0 KNOW BEST?
B B B B B B B B B B B B B B B B B B B B B B B B B B B B B B B B B B B B B B B B B B B B B B B B 1 1 1 English
C C C C C C C C C C C C C C C C C C C C C C C C C C C C C C C C C C C C C C C C C C C C C C C C 2 2 2 English and another language
about the same
D D D D D D D D D D D D D D D D D D D D D D D D D D D D D D D D D D D D D D D D D D D D D D D D 3 3 3 Another language

E E E E E E E E E E E E E E E E E E E E E E E E E E E E E E E E E E E E E E E E E E E E E E E E 4 4 4

F F F F F F F F F F F F F F F F F F F F F F F F F F F F F F F F F F F F F F F F F F F F F F F F 5 5 5 X. RACIAL/ETHNIC GROUP

G G G G G G G G G G G G G G G G G G G G G G G G G G G G G G G G G G G G G G G G G G G G G G G G 6 6 6 Please answer both questions about


Hispanic origin and about race. For the
H H H H H H H H H H H H H H H H H H H H H H H H H H H H H H H H H H H H H H H H H H H H H H H H 7 7 7
following questions about your identity,
I I I I I I I I I I I I I I I I I I I I I I I I I I I I I I I I I I I I I I I I I I I I I I I I 8 8 8 Hispanic origins are not races.

J J J J J J J J J J J J J J J J J J J J J J J J J J J J J J J J J J J J J J J J J J J J J J J J 9 9 9
(You may mark all that apply.)
K K K K K K K K K K K K K K K K K K K K K K K K K K K K K K K K K K K K K K K K K K K K K K K K
a. Are you of b. What is your race?
L L L L L L L L L L L L L L L L L L L L L L L L L L L L L L L L L L L L L L L L L L L L L L L L Hispanic,
Latino, or
American Indian or
M M M M M M M M M M M M M M M M M M M M M M M M M M M M M M M M M M M M M M M M M M M M M M M M Spanish origin? Alaska Native
N N N N N N N N N N N N N N N N N N N N N N N N N N N N N N N N N N N N N N N N N N N N N N N N No, not of Asian (including Indian
Hispanic, Latino, subcontinent and
O O O O O O O O O O O O O O O O O O O O O O O O O O O O O O O O O O O O O O O O O O O O O O O O or Spanish origin Philippines origin)

P P P P P P P P P P P P P P P P P P P P P P P P P P P P P P P P P P P P P P P P P P P P P P P P Yes, Cuban Black or African American


(including Africa and
Q Q Q Q Q Q Q Q Q Q Q Q Q Q Q Q Q Q Q Q Q Q Q Q Q Q Q Q Q Q Q Q Q Q Q Q Q Q Q Q Q Q Q Q Q Q Q Q Yes, Mexican Afro-Caribbean origin)

R R R R R R R R R R R R R R R R R R R R R R R R R R R R R R R R R R R R R R R R R R R R R R R R Yes, Puerto Rican Native Hawaiian or other


Pacific Islander
S S S S S S S S S S S S S S S S S S S S S S S S S S S S S S S S S S S S S S S S S S S S S S S S Yes, another White (including Middle
Hispanic, Latino, Eastern origin)
T T T T T T T T T T T T T T T T T T T T T T T T T T T T T T T T T T T T T T T T T T T T T T T T or Spanish origin

U U U U U U U U U U U U U U U U U U U U U U U U U U U U U U U U U U U U U U U U U U U U U U U U

V V V V V V V V V V V V V V V V V V V V V V V V V V V V V V V V V V V V V V V V V V V V V V V V Y. PARENTAL EDUCATION LEVEL


In the first column, indicate the highest level of education of
W W W W W W W W W W W W W W W W W W W W W W W W W W W W W W W W W W W W W W W W W W W W W W W W
one parent/guardian, and indicate whether this is your mother/
X X X X X X X X X X X X X X X X X X X X X X X X X X X X X X X X X X X X X X X X X X X X X X X X female guardian or father/male guardian. Then, if applicable,
indicate the highest level of education of your other parent/
Y Y Y Y Y Y Y Y Y Y Y Y Y Y Y Y Y Y Y Y Y Y Y Y Y Y Y Y Y Y Y Y Y Y Y Y Y Y Y Y Y Y Y Y Y Y Y Y guardian in the second column, and indicate whether this is
Z Z Z Z Z Z Z Z Z Z Z Z Z Z Z Z Z Z Z Z Z Z Z Z Z Z Z Z Z Z Z Z Z Z Z Z Z Z Z Z Z Z Z Z Z Z Z Z your mother/female guardian or father/male guardian.

0 0 0 0 0 0 0 0 0 0 0 0 0 0 0 0 0 0 0 0 0 0 0 0 0 0 STATE MI NY VT 0 0 0 0 0 0 0 0 0 Mother or female guardian

1 1 1 1 1 1 1 1 1 1 1 1 1 1 1 1 1 1 1 1 1 1 1 1 1 1 AK HI MN OH WA 1 1 1 1 1 1 1 1 1 Father or male guardian

2 2 2 2 2 2 2 2 2 2 2 2 2 2 2 2 2 2 2 2 2 2 2 2 2 2 AL IA MO OK WI 2 2 2 2 2 2 2 2 2 Grade school

3 3 3 3 3 3 3 3 3 3 3 3 3 3 3 3 3 3 3 3 3 3 3 3 3 3 AR ID MS OR WV 3 3 3 3 3 3 3 3 3 Some high school

4 4 4 4 4 4 4 4 4 4 4 4 4 4 4 4 4 4 4 4 4 4 4 4 4 4 AZ IL MT PA WY 4 4 4 4 4 4 4 4 4 High school diploma or equivalent

5 5 5 5 5 5 5 5 5 5 5 5 5 5 5 5 5 5 5 5 5 5 5 5 5 5 CA IN NC RI Puerto 5 5 5 5 5 5 5 5 5 Vocational or trade school


Rico
6 6 6 6 6 6 6 6 6 6 6 6 6 6 6 6 6 6 6 6 6 6 6 6 6 6 CO KS ND SC 6 6 6 6 6 6 6 6 6 Some college

7 7 7 7 7 7 7 7 7 7 7 7 7 7 7 7 7 7 7 7 7 7 7 7 7 7 CT KY NE SD AA 7 7 7 7 7 7 7 7 7 Associate or two-year degree

8 8 8 8 8 8 8 8 8 8 8 8 8 8 8 8 8 8 8 8 8 8 8 8 8 8 DC LA NH TN AE 8 8 8 8 8 8 8 8 8 Bachelor’s or four-year degree

9 9 9 9 9 9 9 9 9 9 9 9 9 9 9 9 9 9 9 9 9 9 9 9 9 9 DE MA NJ TX AP 9 9 9 9 9 9 9 9 9 Some graduate or professional school

/ / / / / / / / / / / / / / / / / / / / / / / / FL MD NM UT Other / / / / / / / Graduate or professional degree


GA ME NV VA T. STUDENT IDENTIFIER (Student ID Number)
S. FOR STUDENTS OUTSIDE If your address does not fit in the spaces provided in Item R, fill in as
THE UNITED STATES ONLY many circles as you can, then fill in the circle in Item S and print the remainder of your address in the spaces provided.
Address City State or Province Country ZIP or Postal Code

By providing your email address, you are granting the College Board
U. EMAIL ADDRESS permission to use your email address in accordance with the policies
in the 2017-18 Bulletin for AP Students and Parents.

Downloaded by Loadster (kadenpizza66@gmail.com)


lOMoARcPSD|34158146

Section I: Multiple-Choice
and Short-Answer Questions

This is the multiple-choice and short-answer section of the 2018 AP Exam.


It includes cover material and other administrative instructions
to help familiarize students with the mechanics of the exam.
(Note that future exams may differ in look from the following content.)

Downloaded by Loadster (kadenpizza66@gmail.com)


lOMoARcPSD|34158146

®
AP United States History Exam
SECTION I, Part A: Multiple Choice 2018
DO NOT OPEN THIS BOOKLET UNTIL YOU ARE TOLD TO DO SO.

At a Glance Instructions
Section I, Part A of this exam contains 55 multiple-choice questions. Fill in only the circles
Time for numbers 1 through 55 on your multiple-choice answer sheet. Because this section
55 minutes offers only four answer options for each question, do not mark the (E) answer circle for
Number of Questions any question.
55
Percent of Total Score Indicate all of your answers to the multiple-choice questions on the multiple-choice answer
40% sheet. No credit will be given for anything written in this exam booklet, but you may use
Writing Instrument the booklet for notes or scratch work. After you have decided which of the suggested
Pencil required
answers is best, completely fill in the corresponding circle on the multiple-choice answer
sheet. Give only one answer to each question. If you change an answer, be sure that the
previous mark is erased completely. Here is a sample question and answer.

Use your time effectively, working as quickly as you can without losing accuracy. Do not
spend too much time on any one question. Go on to other questions and come back to
the ones you have not answered if you have time. It is not expected that everyone will
know the answers to all of the multiple-choice questions.
Your total score on the multiple-choice section is based only on the number of questions
answered correctly. Points are not deducted for incorrect answers or unanswered
questions.

SECTION I, Part B: Short Answer

At a Glance Instructions
For Section I, Part B of this exam, answer Question 1 and Question 2 and either
Time Question 3 or Question 4. Write your responses in the Section I, Part B: Short-Answer
40 minutes Response booklet. You must write your response to each question on the lined page
Number of Questions designated for that response. Each response is expected to fit within its designated page.
3 Fill in the circle on the Section I, Part B: Short-Answer Response booklet indicating
Percent of Total Score whether you answered Question 3 or Question 4. Failure to do so may delay your score.
20%
Writing Instrument
Pen with black or dark
blue ink
Questions 1 and 2
Mandatory Form I
Question 3 or 4 Form Code 4OBP4-S
Choose One Question

07
Downloaded by Loadster (kadenpizza66@gmail.com)
lOMoARcPSD|34158146

The inclusion of source material in this exam is not intended as


an endorsement by the College Board or ETS of the content, ideas,
or values expressed in the material. The material has been selected
by the history faculty who serve on the AP United States History
Development Committee. In their judgment, the material printed here
reflects various aspects of the course of study on which this exam is
based and is therefore appropriate to use to measure the skills and
knowledge of this course.

-2-

Downloaded by Loadster (kadenpizza66@gmail.com)


lOMoARcPSD|34158146

UNITED STATES HISTORY


SECTION I, Part A
Time—55 minutes
55 Questions

Directions: Each of the questions or incomplete statements is followed by four suggested answers or completions.
Select the one that is best in each case and then fill in the corresponding circle on the answer sheet.

Questions 1 and 2 refer to the following excerpt.

“We have as yet no certaine proofe or experience concerning the vertues of . . . Corne, although the . . .
Indians . . . are constrained to make a virtue of necessitie, and think it a good food: whereas we may
easily judge that it nourisheth but little, and is of a hard . . . digestion, a more convenient food for swine than
for men.”
John Gerard, English botanist, The Herball or Generall Historie of
Plantes, 1597

1. The development that brought “corne” to the 2. Gerard’s description of “corne” in the excerpt
attention of botanists such as Gerard is best best reflects which of the following?
known as the
(A) Curiosity about American Indian ways
(A) Columbian Exchange of life
(B) encomienda system (B) Assumptions about the superiority of
European culture
(C) trans-Atlantic slave trade
(C) Inability to find uses for North American
(D) Enlightenment
crops
(D) Concerns about economic threats posed by
North American imports

-3- GO ON TO THE NEXT PAGE.

Downloaded by Loadster (kadenpizza66@gmail.com)


lOMoARcPSD|34158146

Questions 3 - 5 refer to the following excerpt.

“Various are the reports and conjectures of the causes of the present Indian war. Some impute it to an
imprudent zeal in the magistrates of Boston to christianize those heathen before they were civilized and
enjoining them the strict observation of their laws. . . . Some believe there have been vagrant and Jesuitical
priests, who have made it their business, for some years past, to go from Sachem to Sachem, to exasperate the
Indians against the English and to bring them into a confederacy, and that they were promised supplies from
France and other parts to extirpate [eradicate] the English nation out of the continent of America.”

Edward Randolph, report of King Philip’s War (Metacom’s War) in


New England, 1676

3. The confederacy formed to “exasperate the 5. Compared with French and Spanish interactions
Indians against the English” was motivated with American Indians, English interaction with
primarily by which of the following? American Indians more often promoted
(A) Fraudulent trade deals between the leaders (A) respect for political alliances
of Plymouth Colony and the
(B) cultural blending
Wampanoags
(C) separation between the groups
(B) Dispossession of Wampanoag land and
threats to their sovereignty (D) assimilation of Americans Indians into
colonial societies
(C) Forced religious conversion of
Wampanoags by Puritan missionaries
(D) Intermarriage with the English, which
threatened Wampanoag cultural
independence

4. Which of the following best characterizes


relations between the English and American
Indians in New England following
Metacom’s War?

(A) Peaceful accommodation between both


groups
(B) Religious freedom for the English and
American Indians
(C) Dramatic decline and dispersion of the
American Indian population
(D) Recognition of American Indian property
and land rights

-4- GO ON TO THE NEXT PAGE.

Downloaded by Loadster (kadenpizza66@gmail.com)


lOMoARcPSD|34158146

Questions 6 and 7 refer to the following excerpt.

“That a British and American legislature, for regulating the administration of the general affairs of America,
be proposed and established in America, including all the said colonies; within, and under which government,
each colony shall retain its present constitution, and powers of regulating and governing its own internal
police, in all cases whatsoever.

“That the said government be administered by a President General, to be appointed by the King and a Grand
Council, to be chosen by the representatives of the people of the several colonies, in their respective
assemblies, once in every three years.”

Joseph Galloway, “A Plan of a Proposed Union Between Great Britain and


the Colonies,” proposal debated by the First Continental Congress, 1774

6. The excerpt most strongly suggests that in 1774 7. The key concern that Galloway’s plan was
which of the following was correct? designed to address was the
(A) The American colonists were united in (A) lack of American representation in the
their desire for independence from Great British Parliament
Britain.
(B) demand for back pay for veterans of the
(B) The American colonists felt the need to French and Indian War (Seven
unite against the threat from American Years’ War)
Indians.
(C) complaint about high tariffs imposed on
(C) Some members of the First Continental colonial merchants for exports to Great
Congress sought a compromise between Britain
submission to British authority and
(D) continuing security threat posed by the
independence.
French in Canada
(D) Some members of the First Continental
Congress thought it would be relatively
easy to unify the colonies and win
independence from Great Britain.

-5- GO ON TO THE NEXT PAGE.

Downloaded by Loadster (kadenpizza66@gmail.com)


lOMoARcPSD|34158146

Questions 8 - 10 refer to the following excerpt.

“The petition of a great number of blacks detained in a state of slavery in the bowels of a free and Christian
country humbly showeth that . . . they have in common with all other men a natural and inalienable right to
that freedom which the Great Parent of the Universe has bestowed equally on all mankind and which they
have never forfeited by any compact or agreement whatever. . . .

“[E]very principle from which America has acted in the course of their unhappy difficulties with Great
Britain pleads stronger than a thousand arguments in favor of your petitioners. They therefore humbly
beseech your honors to give this petition its due weight and consideration and cause an act of the legislature
to be passed whereby they may be restored to the enjoyments of that which is the natural right of all men.”

Petition for freedom to the Massachusetts Council and the House of


Representatives for the State of Massachusetts, January 1777

8. The ideas expressed in the excerpt contributed 10. Which of the following most likely helped to
most directly to which of the following? prompt the petition in the excerpt?

(A) The extension of voting rights to African (A) American colonists’ declaration of
Americans in the North independence from Britain
(B) The end of the trans-Atlantic slave trade (B) British promises of land for service in the
American Revolution
(C) The mass migration of African Americans
from the South to the North (C) The passage of the Stamp Act, a tax
imposed on the American colonies by the
(D) The adoption of plans for gradual
British Parliament
emancipation in the North
(D) The proliferation of religious revivals in
9. Which of the following developments from the the eighteenth century, commonly
1800s emerged from ideas most similar to those referred to as the Great Awakening
expressed in the excerpt?
(A) Campaigns by moral reformers to promote
temperance
(B) Efforts by American Indians to achieve
political sovereignty through treaties with
the United States government
(C) The ratification of the Fourteenth
Amendment to the United States
Constitution
(D) The passage of legislation by southern
states intended to nullify federal laws

-6- GO ON TO THE NEXT PAGE.

Downloaded by Loadster (kadenpizza66@gmail.com)


lOMoARcPSD|34158146

Questions 11 - 13 refer to the following excerpt.

“We, therefore, the people of the State of South Carolina, in convention assembled, do declare and ordain . . .
that the several acts and parts of acts of the Congress of the United States, purporting to be laws for the
imposing of duties and imposts on the importation of foreign commodities . . . are unauthorized by the
Constitution of the United States, and violate the true meaning and intent thereof and are null, void, and no
law, nor binding upon this State. . . .”

South Carolina Ordinance of Nullification, 1832

11. The ideas expressed in the excerpt emerged 13. Arguments similar to those expressed in the
most directly from a larger intellectual debate excerpt were later employed to justify which of
over the the following?

(A) balance between individual freedom and (A) The entry into the Mexican-American War
public order
(B) The passage of the Kansas-Nebraska Act
(B) expansion of slavery into the western
(C) The secession of most Southern states
territories
(D) The ratification of the Fourteenth
(C) priorities of United States foreign policy
Amendment
(D) relationship between the federal
government and the states

12. The excerpt most directly expresses an


economic perspective that

(A) prioritized regional interests


(B) discouraged international trade
(C) sought to protect United States
manufacturing
(D) supported the interests of organized labor
unions

-7- GO ON TO THE NEXT PAGE.

Downloaded by Loadster (kadenpizza66@gmail.com)


lOMoARcPSD|34158146

Questions 14 - 16 refer to the following excerpt.

“Resolved, That woman is man’s equal. . . .

“Resolved, That woman has too long rested satisfied in the circumscribed limits which corrupt customs . . .
have marked out for her, and that it is time she should move in the enlarged sphere . . . assigned her.

“Resolved, That it is the duty of the women of this country to secure to themselves their sacred right to the
elective franchise.

“Resolved, . . . That, being invested by the Creator with the same capabilities, and the same consciousness of
responsibility for their exercise, it is demonstrably the right and duty of woman, equally with man, to
promote every righteous cause, by every righteous means.”

Declaration of Sentiments and Resolutions (Seneca Falls Convention), 1848

14. The language and themes of the excerpt were 16. In the decades following the Civil War, the
most directly inspired by the woman’s rights movement that began at Seneca
Falls focused its energies most strongly on
(A) Articles of Confederation
(A) achieving the right to vote
(B) Declaration of Independence
(B) receiving equal pay for equal work
(C) Northwest Ordinance
(C) ending domestic violence
(D) United States Constitution
(D) gaining equal access to higher education
15. Which other “righteous cause” would
participants in the Seneca Falls Convention have
been most likely to support?
(A) Expansionism
(B) Nativism
(C) Abolitionism
(D) Conservationism

-8- GO ON TO THE NEXT PAGE.

Downloaded by Loadster (kadenpizza66@gmail.com)


lOMoARcPSD|34158146

Questions 17 - 19 refer to the following excerpt.

“For a few years in the 1850s, ethnic conflict among whites rivaled sectional conflict as a major political
issue. The immediate origins of this phenomenon lay in the sharp increase of immigration after 1845. . . . The
average quadrupled in the 1830s. But even this paled in comparison with the immigration of the late
1840s. . . . During the decade 1846–1855, more than three million immigrants entered the United
States—equivalent to 15 percent of the 1845 population. This was the largest proportional increase in the
foreign-born population for any ten-year period in American history. . . . Equal in significance to the increase
in the foreign-born population were changes in its composition.”

James M. McPherson and James K. Hogue, historians, Ordeal By Fire: The


Civil War and Reconstruction, 2010

17. Which of the following most directly 19. The conflict described in the excerpt is most
contributed to “the sharp increase of similar to conflict in what other period?
immigration after 1845” referenced in the
excerpt? (A) The period from after the Seven Years’ War
through the 1760s
(A) The Second Great Awakening
(B) The period from after the War of 1812
(B) Crop failures and revolutions in Europe through the 1820s
(C) Removal of American Indians from the (C) The period from after the First World War
Southeast through the 1920s
(D) Tariff policies during Andrew Jackson’s (D) The period from after the Second World
administration War through the 1950s

18. Which of the following could best be used as


evidence to support the argument in the excerpt
that “ethnic conflict among whites rivaled
sectional conflict as a major political issue” of
the period?

(A) Growing concern about the political and


cultural influence of Catholic immigrants
(B) Growing fear of political radicalism among
southern and eastern European
immigrants
(C) Increasing cultural influence of European
Romanticism in the United States
(D) Increasing support for the antislavery cause
among the immigrant community

-9- GO ON TO THE NEXT PAGE.

Downloaded by Loadster (kadenpizza66@gmail.com)


lOMoARcPSD|34158146

Questions 20 - 22 refer to the following excerpt.

“It is for us the living, rather, to be dedicated here to the unfinished work which they who fought here have
thus far so nobly advanced. It is rather for us to be here dedicated to the great task remaining before us—that
from these honored dead we take increased devotion to that cause for which they gave the last full measure of
devotion—that we here highly resolve that these dead shall not have died in vain—that this nation, under
God, shall have a new birth of freedom—and that government of the people, by the people, for the people,
shall not perish from the earth.”

Abraham Lincoln, Gettysburg Address, November 1863

20. After 1863, which of the following most 22. Lincoln’s main purpose in the excerpt was to
fulfilled the “new birth of freedom” that the
excerpt refers to? (A) advocate racial equality
(B) encourage the punishment of the South
(A) Ratification of the Thirteenth, Fourteenth,
and Fifteenth Amendments (C) propose expanded democratic voting rights
(B) The compromise that resolved the election (D) gain continued support for the war effort
of 1876
(C) Establishment of the Ku Klux Klan and
similar organizations
(D) Supreme Court rulings such as Plessy v.
Ferguson

21. Which of the following most directly


contributed to the conflict referred to in the
excerpt?
(A) Disputes over taxation and representation
(B) Tensions between isolationism and
international engagement
(C) Disagreements over whether to allow
slavery in new territories
(D) Debates about the role of religion in
society and government

-10- GO ON TO THE NEXT PAGE.

Downloaded by Loadster (kadenpizza66@gmail.com)


lOMoARcPSD|34158146

Questions 23 - 25 refer to the following excerpt.

“In August 1865, the photographer Marcus Ormsbee . . . took a formal portrait of several groups of craft
workers in their different shops. . . . At the center of the photograph, at Outcault’s carpentry shop, stands the
conventional artisan trio of master, journeyman, and apprentice, still at the heart of the city’s workshop
world—yet class differences mark these craftsmen’s every feature. . . . Brooding above everyone, a new brick
manufactory seals off its employees from the street and from public view. Small shop and large enterprise
converge; New York remains a blend of old and new.”

Sean Wilentz, historian, Chants Democratic, 1984

23. Which of the following most directly led to the 25. Which of the following is one important
changes described in the excerpt? continuity in urban life in the United States
throughout the nineteenth century?
(A) Innovative uses of photography to achieve
social change (A) Anarchism and similar radical ideologies
attracted many workers.
(B) Technological innovations in the
production of goods (B) Settlement houses assisted immigrants
with adapting to life in the United States.
(C) The economic hardship resulting from
years of civil war in the United States (C) Workers and employers
consistently maintained amicable
(D) The end of the apprenticeship system in
relationships.
craft work
(D) Immigrants formed an important part of
24. The conditions described in the excerpt most the manufacturing workforce.
directly contributed to the

(A) decrease in tariffs for manufactured goods


(B) growth of an organized labor movement
(C) expansion of government regulation of
industry
(D) loss of class distinctions in the United
States

-11- GO ON TO THE NEXT PAGE.

Downloaded by Loadster (kadenpizza66@gmail.com)


lOMoARcPSD|34158146

Questions 26 - 28 refer to the following 1871 image.

William M. “Boss” Tweed: “As long as I count the votes, what are you going to
do about it?”

Fotosearch/Getty Images

-12- GO ON TO THE NEXT PAGE.

Downloaded by Loadster (kadenpizza66@gmail.com)


lOMoARcPSD|34158146

26. The image was created most directly in 28. During the late nineteenth century, politicians
response to such as the one depicted in the image most
likely would have opposed which of the
(A) social reform efforts by settlement-house following?
workers
(A) Social services provided to immigrants by
(B) poll taxes and literacy tests that blocked
local politicians
African Americans from voting
(B) Granting American Indians full citizenship
(C) the power gained by urban political
and voting rights
machines
(C) The expansion of the transcontinental
(D) the exclusion of women from voting
railroad system

27. Which of the following changes to the United (D) Calls for reforms to local and state
States during the nineteenth century most governments
directly contributed to the development depicted
in the image?

(A) The rapid growth of cities


(B) The dismantling of the national bank
(C) The acquisition of territory in the West
(D) The rise of the People’s (Populist) Party

-13- GO ON TO THE NEXT PAGE.

Downloaded by Loadster (kadenpizza66@gmail.com)


lOMoARcPSD|34158146

Questions 29 - 32 refer to the following 1901 image.


Columbia’s Easter Bonnet

Courtesy of the Library of Congress

-14- GO ON TO THE NEXT PAGE.

Downloaded by Loadster (kadenpizza66@gmail.com)


lOMoARcPSD|34158146

29. The ideas addressed in the image most directly 31. The ideas expressed through the image reveal
relate to that in 1901, which of the following was most
true of the United States?
(A) opposition to the United States collective
security arrangement with Western (A) Theories of survival of the fittest had been
Europe widely rejected by the public.
(B) the fear of increased militarism among (B) Efforts to spread democracy overseas had
European and Pacific powers been largely peaceful.
(C) the increased visibility of women’s (C) Interventionism had become a more
organizations that criticized imperialism prevalent feature of foreign policy.
(D) debates about United States acquisition of (D) Women were encouraged to join the armed
overseas territories forces.

30. Which of the following most directly led to the 32. The developments referenced by the image most
circumstances illustrated by the image? directly contributed to United States
involvement in
(A) The United States victory in the
Spanish-American War (A) competition with Russia in opening trade
with Japan
(B) The Supreme Court ruling in Plessy v.
Ferguson (B) the suppression of an independence
movement in the Philippines
(C) The United States contributions to the
Allied victory in the First World War (C) acquisition of territory from Mexico
through the Treaty of Guadalupe Hidalgo
(D) The ratification of the Nineteenth
Amendment to the Constitution (D) an international monetary system that
supported free trade between nations

-15- GO ON TO THE NEXT PAGE.

Downloaded by Loadster (kadenpizza66@gmail.com)


lOMoARcPSD|34158146

Questions 33 - 35 refer to the following excerpt.

“The National Progressive Party, committed to the principle of government by a self-controlled democracy
expressing its will through representatives of the people, pledges itself to secure such alterations in the
fundamental law of the several States and of the United States as shall insure the representative character of
the government.”
Progressive Party Platform, 1912

33. The excerpt suggests that Progressives in the 35. Which of the following groups is most credited
early twentieth century most typically sought to with advancing Progressivism?

(A) challenge political inequality (A) Anarchist activists


(B) advocate a return to agrarianism (B) Recent immigrants
(C) justify the inequality of wealth (C) Agricultural workers
(D) oppose United States imperialism (D) Middle-class women

34. Progressivism, as described in the excerpt, has


the most in common with which of the
following later domestic policy initiatives?
(A) President Lyndon Johnson’s protection of
voting rights during the Great Society
(B) President Ronald Reagan’s deregulation of
industries
(C) President Bill Clinton’s changes to welfare
policy
(D) President Herbert Hoover’s support for
increased tariffs during the Great
Depression

-16- GO ON TO THE NEXT PAGE.

Downloaded by Loadster (kadenpizza66@gmail.com)


lOMoARcPSD|34158146

Questions 36 - 38 refer to the following excerpt.

“Beginning in the 1930s and lasting into the 1950s, black Chicago experienced a cultural renaissance. . . .
Chicago had become a major destination for black southern migrants. . . . It was also an urban industrial
center. This fact gave a unique working-class and internationalist perspective to the cultural work that would
take place there. . . .

“A desire to live freely in ‘the metropolis’ continued to characterize the aspirations of migrants as
second-wave Chicago migrants arrived. . . . The 1930s and 1940s witnessed a resurgence of black
working-class political radicalism that was captured and reflected in the expressive visual and literary
productions of Chicago Black Renaissance artists.”

Darlene Clark Hine, historian, The Black Chicago Renaissance, 2012

36. The “political radicalism” referred to in the 38. Which of the following most likely contributed
excerpt was most directly a response to the to the decline of the “political radicalism”
experience of referred to in the excerpt?

(A) desegregation (A) The drafting of young men to fight in


Vietnam
(B) economic depression
(B) Racial discrimination in the United States
(C) mass consumption
armed forces during the Second
(D) nativism World War
(C) Anticommunist measures during the
37. Before moving to Chicago, the people described Cold War
in the excerpt most likely were engaged in
which of the following? (D) Neglect of Black artists and culture during
the New Deal
(A) Sharecropping or tenant farming
(B) Gang labor on the railways
(C) Mining and other extractive industries
(D) Manufacturing in urban factories

-17- GO ON TO THE NEXT PAGE.

Downloaded by Loadster (kadenpizza66@gmail.com)


lOMoARcPSD|34158146

Questions 39 - 41 refer to the following excerpt.

“In the future days, which we seek to make secure, we look forward to a world founded upon four essential
human freedoms.

“The first is freedom of speech and expression—everywhere in the world.

“The second is freedom of every person to worship God in his own way—everywhere in the world.

“The third is freedom from want—which, translated into world terms, means economic understandings which
will secure to every nation a healthy peacetime life for its inhabitants—everywhere in the world.

“The fourth is freedom from fear—which, translated into world terms, means a world-wide reduction of
armaments to such a point and in such a thorough fashion that no nation will be in a position to commit an
act of physical aggression against any neighbor—anywhere in the world.”

President Franklin D. Roosevelt, State of the Union address, January 1941

39. Roosevelt’s speech was most likely intended to 41. Which of the following most immediately
increase public support for increased enthusiasm in the United States for
upholding the freedoms outlined in the excerpt?
(A) extending New Deal programs to guarantee
unemployment benefits (A) The passage of a federal law providing
veterans with financial aid in housing and
(B) integrating the United States armed forces
education
(C) aiding the Allies in Europe during the
(B) The successful detonation of an atomic
Second World War
bomb as part of the Manhattan Project
(D) enhancing rights protected by the United
(C) The growing number of women working in
States Constitution
defense manufacturing

40. Which of the following most directly (D) The attack on the United States at
contributed to Roosevelt’s view that “freedom Pearl Harbor
from want” was necessary?
(A) The inspiration of Woodrow Wilson’s
vision for an international democratic
order
(B) The experience of domestic and global
economic upheavals during the 1930s
(C) The memory of federal agencies violating
some Americans’ civil liberties in the
1920s
(D) The fear of the Soviet threat to spread
communism to nations in the West

-18- GO ON TO THE NEXT PAGE.

Downloaded by Loadster (kadenpizza66@gmail.com)


lOMoARcPSD|34158146

Questions 42 - 44 refer to the following excerpt.

“[After the Second World War, Americans] wanted . . . a secure country. Security would enable them to take
advantage of the fruits of prosperity and peace. . . . And so they adhered to an overarching principle that
would guide them in their personal and political lives: containment. . . . Domestic containment was bolstered
by a powerful political culture that rewarded its adherents and marginalized its detractors. . . . [C]ontainment
aptly describes the way in which public policy, personal behavior, and even political values were focused on
the home. . . . Vast numbers of American women and men during the early years of the cold war . . . got
married, moved to the suburbs, and had babies. . . . [F]ew were willing to give up the rewards of conforming
for the risks of resisting the domestic path.”

Elaine Tyler May, historian, Homeward Bound: American Families in the


Cold War Era, 1988

42. The rise of what the excerpt describes as 44. A desire for greater national security in the
“domestic containment” most directly United States emerged immediately after the
contributed to which of the following Second World War because of fear of which of
characteristics of United States society during the following?
the period?
(A) The spread of international communism
(A) White Southern resistance to school
(B) Decolonization in Africa and Asia
integration
(C) The emergence of détente
(B) Greater cultural homogeneity
(D) Oil crises in the Middle East
(C) Evangelical Protestant churches’ increased
political engagement
(D) The popularity of liberal politics

43. The pattern described in the excerpt of


Americans who “got married, moved to the
suburbs, and had babies” most directly
contributed to which of the following long-term
developments?
(A) Increased immigration from abroad
(B) Greater concern about the environment
(C) Wage stagnation for the working and
middle classes
(D) The rise of youth rebellion through the
counterculture

-19- GO ON TO THE NEXT PAGE.

Downloaded by Loadster (kadenpizza66@gmail.com)


lOMoARcPSD|34158146

Questions 45 - 48 refer to the following graph.

United States Bureau of Labor Statistics

45. The overall trend from 1960 to 1970 depicted on 47. The overall trend from 1980 to 2010 depicted on
the graph resulted from all of the following the graph was most directly caused by the
EXCEPT the
(A) reliance of the United States on fossil fuels
(A) growth of corporations
(B) increasing integration of the United States
(B) continuation of federal investment into the world economy
(C) adoption of digital technologies (C) reforms to the United States social welfare
system
(D) development of new consumer products
(D) boom in the United States financial and
46. A development related to the overall trend from stock markets
1960 to 1980 depicted on the graph was the
48. The overall trend from 1980 to 2010 depicted on
(A) increase in public concern regarding air the graph most directly led to a decline in
and water quality
(A) income inequality
(B) widespread replacement of male factory
workers with female workers (B) immigration from abroad
(C) creation of the first trusts and monopolies (C) concern about the environment
(D) ban of free-trade agreements between (D) union membership
North American countries

-20- GO ON TO THE NEXT PAGE.

Downloaded by Loadster (kadenpizza66@gmail.com)


lOMoARcPSD|34158146

Questions 49 and 50 refer to the following excerpt.

“We are people of this generation, bred in at least modest comfort, housed now in universities. . . . Freedom
and equality for each individual, government of, by, and for the people—these American values we found
good. . . . As we grew, however, our comfort was penetrated by events too troubling to dismiss. . . . The
declaration ‘all men are created equal’ rang hollow before the facts of Negro life. . . . The proclaimed
peaceful intentions of the United States contradicted its economic and military investments in the Cold War
status quo. . . . America rests in national stalemate, . . . its democratic system apathetic and manipulated
rather than ‘of, by, and for the people.’”

Students for a Democratic Society, “The Port Huron Statement,” 1962

49. Which of the following most directly 50. Based on the excerpt, Students for a Democratic
contributed to the sentiments expressed in the Society would most likely support
excerpt?
(A) greater global economic integration under
(A) Concerns about the declining role of the United States leadership
United States in Western Europe
(B) expanded attempts to repress communists
(B) The increasing use of federal power to in the United States
combat racial discrimination
(C) a decrease in United States intervention
(C) Social inequalities exposed during the in Asia
Civil Rights movement
(D) reduced spending for government social
(D) Support for the equality of gays and programs
lesbians

-21- GO ON TO THE NEXT PAGE.

Downloaded by Loadster (kadenpizza66@gmail.com)


lOMoARcPSD|34158146

Questions 51 - 53 refer to the following excerpt.

“I will not accept the excuse that the federal government has grown so big and powerful that it is beyond the
control of any president, any administration or Congress. We are going to put an end to the notion that the
American taxpayer exists to fund the federal government. The federal government exists to serve the
American people. . . . We are taxing ourselves into economic exhaustion and stagnation, crushing our ability
and incentive to save, invest, and produce. This must stop.”

Ronald Reagan, speech accepting the Republican Party’s nomination for


president, 1980

51. Reagan’s administration implemented the ideas 53. The excerpt reflects many Americans’ belief at
expressed in the excerpt by the time that the federal government

(A) increasing defense spending (A) was responsible for building infrastructure
such as roads and schools
(B) reducing regulation of industry
(B) had been unable to solve social and
(C) expanding protection of the environment
economic problems
(D) eliminating major entitlement programs
(C) should vigorously resist communism
such as Medicaid
abroad

52. Which of the following would be most likely to (D) should avoid interfering in moral issues
support the views expressed in the excerpt?
(A) White males in the Sunbelt
(B) Women in the Northeast
(C) African Americans in the Midwest
(D) Latinos in the Southwest

-22- GO ON TO THE NEXT PAGE.

Downloaded by Loadster (kadenpizza66@gmail.com)


lOMoARcPSD|34158146

Questions 54 and 55 refer to the following excerpt.

“What is the phenomenon of globalization . . . ? Fundamentally, it is the closer integration of the countries
and peoples of the world which has been brought about by the enormous reduction of costs of transportation
. . . and the breaking down of artificial barriers to the flow of goods, services, capital, knowledge, and (to a
lesser extent) people across borders.”
Joseph E. Stiglitz, economist, Globalization and Its Discontents, 2002

54. Which of the following earlier trends was most 55. Which of the following contributed most
similar to the pattern described in the excerpt? directly to the trend described in the excerpt?

(A) The development of Atlantic world (A) The implementation of protective tariffs by
commerce in the 1600s and early 1700s trading nations
(B) The appearance of economic cycles in the (B) The spread of computer technology and
early 1800s Internet use
(C) The restrictions on immigration in the (C) The growth of labor unions’ economic
latter half of the 1800s influence
(D) The increases in worldwide tariffs during (D) The increased number and size of cities in
the Great Depression the United States

-23- GO ON TO THE NEXT PAGE.

Downloaded by Loadster (kadenpizza66@gmail.com)


lOMoARcPSD|34158146

END OF PART A
IF YOU FINISH BEFORE TIME IS CALLED,
YOU MAY CHECK YOUR WORK ON PART A.
DO NOT GO ON TO PART B UNTIL YOU ARE TOLD TO DO SO.

_____________________________________________

MAKE SURE YOU HAVE DONE THE FOLLOWING.

• PLACED YOUR AP NUMBER LABEL ON YOUR MULTIPLE-CHOICE ANSWER


SHEET
• WRITTEN AND GRIDDED YOUR AP NUMBER CORRECTLY ON YOUR
MULTIPLE-CHOICE ANSWER SHEET
• TAKEN THE AP EXAM LABEL FROM THE FRONT OF THIS BOOKLET AND PLACED
IT ON YOUR MULTIPLE-CHOICE ANSWER SHEET

-24-

Downloaded by Loadster (kadenpizza66@gmail.com)


lOMoARcPSD|34158146

UNITED STATES HISTORY


SECTION I, Part B
Time—40 minutes

Directions: Answer Question 1 and Question 2. Answer either Question 3 or Question 4.

Write your responses in the Section I, Part B: Short-Answer Response booklet. You must write your response to each
question on the lined page designated for that response. Each response is expected to fit within the space provided.

In your responses, be sure to address all parts of the questions you answer. Use complete sentences; an outline or
bulleted list alone is not acceptable. You may plan your answers in this exam booklet, but no credit will be given for
notes written in this booklet.

“If American sectionalism entered a new phase in 1846, it was neither because North and South clashed for
the first time nor because the issue of slavery for the first time assumed importance. As early as the
Confederation, North and South had been at odds. . . . Once the government under the Constitution went into
effect, bitter sectional conflicts raged. . . . This sectional rivalry tended to become institutionalized in the
opposing [political] organizations. . . . No matter which region embraced nationalism and which
particularism, sectional conflict remained a recurrent phenomenon.”

David M. Potter, The Impending Crisis, 1848–1861, published in 1976

“The Civil War represented an utter and unique breakdown of the normal democratic political process. When
one section of the country refused to accept the decision of a presidential election, secession and the ensuing
war became the great exception to the American political tradition of compromise. The rending of the nation
was the one time that conflict seemed too irrepressible, too fundamental, to be contained within common
consensual boundaries. Because the war was such an anomaly, both participants and later historians have
been fascinated with its causes since the shooting started.”

Michael F. Holt, The Political Crisis of the 1850s, published in 1978

1. Using the excerpts above, answer (a), (b), and (c).

a) Briefly describe ONE major difference between Potter’s and Holt’s historical interpretations of the Civil War.
b) Briefly explain how ONE specific historical event or development during the period 1786–1861 that is not
explicitly mentioned in the excerpts could be used to support Potter’s interpretation.
c) Briefly explain how ONE specific historical event or development during the period 1786–1861 that is not
explicitly mentioned in the excerpts could be used to support Holt’s interpretation.

-26- GO ON TO THE NEXT PAGE.

Downloaded by Loadster (kadenpizza66@gmail.com)


lOMoARcPSD|34158146

Michelin North America, Inc.

2. Using the 1928 image above, answer (a), (b), and (c).

a) Briefly describe ONE perspective about women’s roles during the 1920s expressed through the image.
b) Briefly explain ONE specific historical development or circumstance from 1900 to 1929 that led to changes in
women’s roles during the 1920s such as that depicted in the image.
c) Briefly explain ONE specific historical development in business OR culture that led to the creation and use of
images such as the one above during the 1920s.

-27- GO ON TO THE NEXT PAGE.

Downloaded by Loadster (kadenpizza66@gmail.com)


lOMoARcPSD|34158146

Question 3 or 4
.
Directions: Answer either Question 3 or Question 4.

3. Answer (a), (b), and (c), confining your response to the period 1700 to 1776.

a) Briefly describe ONE specific historical difference in North America between the First Great Awakening and
the Enlightenment.
b) Briefly describe ONE specific historical similarity in North America between the First Great Awakening and
the Enlightenment.
c) Briefly explain ONE specific historical effect in North America of either the First Great Awakening or the
Enlightenment.

4. Answer (a), (b), and (c).

a) Briefly describe ONE specific historical similarity between the United States reasons for involvement in the
Korean War and its reasons for involvement in the Vietnam War.
b) Briefly describe ONE specific historical difference between the United States experience in the Korean War
and in the Vietnam War.
c) Briefly explain ONE specific historical effect of either the Korean War or the Vietnam War on United States
society.

-28- GO ON TO THE NEXT PAGE.

Downloaded by Loadster (kadenpizza66@gmail.com)


lOMoARcPSD|34158146

END OF SECTION I
IF YOU FINISH BEFORE TIME IS CALLED,
YOU MAY CHECK YOUR WORK ON THIS SECTION.
DO NOT GO ON TO SECTION II UNTIL YOU ARE TOLD TO DO SO.

_______________________________________________

MAKE SURE YOU HAVE DONE THE FOLLOWING.

• FILLED IN THE CIRCLE ON THE LAST PAGE OF THE SECTION I, PART B:


SHORT-ANSWER RESPONSE BOOKLET THAT INDICATES WHETHER YOU
ANSWERED SHORT-ANSWER QUESTION 3 OR SHORT-ANSWER QUESTION 4
• PLACED YOUR AP NUMBER LABEL ON YOUR SECTION I, PART B:
SHORT-ANSWER RESPONSE BOOKLET
• COMPLETED THE IDENTIFICATION INFORMATION AS REQUESTED ON THE
FRONT OF THE SECTION I, PART B: SHORT-ANSWER RESPONSE BOOKLET
• USED THE SAME SET OF AP NUMBER LABELS ON ALL AP EXAMS YOU HAVE
TAKEN THIS YEAR

-29-

Downloaded by Loadster (kadenpizza66@gmail.com)


lOMoARcPSD|34158146

Section II: Free-Response Questions

This is the free-response section of the 2018 AP Exam.


It includes cover material and other administrative instructions
to help familiarize students with the mechanics of the exam.
(Note that future exams may differ in look from the following content.)

Downloaded by Loadster (kadenpizza66@gmail.com)


lOMoARcPSD|34158146

®
AP United States History Exam
SECTION II: Free Response 2018
DO NOT OPEN THIS BOOKLET UNTIL YOU ARE TOLD TO DO SO.

At a Glance
Total Time
1 hour and 40 minutes
Number of Questions
2
Percent of Total Score
40%
Writing Instrument
Pen with black or dark
blue ink

Question 1 (DBQ):
Mandatory
Suggested Reading and
Writing Time
1 hour
Reading Period Instructions
15 minutes. Use this
time to read Question 1 The questions for Section II are printed in the orange Questions and Documents booklet.
and to plan your answer. You may use that booklet to organize your answers and for scratch work, but you must
You may begin writing write your answers in this Section II: Free Response booklet. No credit will be given for
your response before any work written in the Questions and Documents booklet.
the reading period is
over. The proctor will announce the beginning and end of the reading period. You are advised
Suggested Writing Time to spend the 15-minute period reading the question and planning your answer to
45 minutes
Question 1, the document-based question (DBQ). If you have time, you may also read
Percent of Total Score
25%
Questions 2, 3, and 4. You may begin writing your responses before the reading period is
over.
Question 2,
Section II of this exam requires answers in essay form. Write clearly and legibly. Circle the
3, or 4: Choose number of the question you are answering at the top of each page in this booklet. Begin
One Question each answer on a new page. Do not skip lines. Cross out any errors you make; crossed-out
Answer Question 2, work will not be scored.
Question 3, or
Question 4 Manage your time carefully. The proctor will announce the suggested time for each part,
Suggested Writing Time but you may proceed freely from one part to the next. Go on to Question 2, 3, or 4 if you
40 minutes finish Question 1 early. You may review your responses if you finish before the end of the
Percent of Total Score exam is announced.
15%
After the exam, you must apply the label that corresponds to the question you
answered. For example, if you answered Question 2, apply the label to the front
cover. Failure to do so may delay your score.

Form I
Form Code 4OBP4-S

07
Downloaded by Loadster (kadenpizza66@gmail.com)
lOMoARcPSD|34158146

THIS PAGE MAY BE USED FOR PLANNING YOUR ANSWERS. NOTES WRITTEN IN THIS ORANGE
BOOKLET WILL NOT BE SCORED. WRITE ALL YOUR RESPONSES IN THE SECTION II BOOKLET.

SECTION II QUESTIONS AND DOCUMENTS BEGIN ON PAGE 4.

-3-

Downloaded by Loadster (kadenpizza66@gmail.com)


lOMoARcPSD|34158146

UNITED STATES HISTORY


SECTION II
Total Time—1 hour and 40 minutes

Question 1 (Document-Based Question)


Suggested reading and writing time: 1 hour

It is suggested that you spend 15 minutes reading the documents and 45 minutes writing your response.
Note: You may begin writing your response before the reading period is over.

Directions: Question 1 is based on the accompanying documents. The documents have been edited for the purpose
of this exercise.

In your response you should do the following.

• Respond to the prompt with a historically defensible thesis or claim that establishes a line of reasoning.
• Describe a broader historical context relevant to the prompt.
• Support an argument in response to the prompt using at least six documents.
• Use at least one additional piece of specific historical evidence (beyond that found in the documents) relevant to
an argument about the prompt.
• For at least three documents, explain how or why the document’s point of view, purpose, historical situation,
and/or audience is relevant to an argument.
• Use evidence to corroborate, qualify, or modify an argument that addresses the prompt.

-4- GO ON TO THE NEXT PAGE.

Downloaded by Loadster (kadenpizza66@gmail.com)


lOMoARcPSD|34158146

1. Evaluate the extent of change in United States foreign policy in the period 1783 to 1828.

Document 1

Source: President George Washington, Neutrality Proclamation, April 1793.

Whereas it appears that a state of war exists between Austria, Prussia, Sardinia, Great Britain, and the United
Netherlands, of the one part, and France on the other; and the duty and interest of the United States require,
that they should with sincerity and good faith adopt and pursue a conduct friendly and impartial toward the
belligerent powers:

I have therefore thought fit by these presents to declare the disposition of the United States to observe the
conduct aforesaid towards those powers respectively; and to exhort and warn the citizens of the United States
carefully to avoid all acts and proceedings whatsoever, which may in any manner tend to contravene such
disposition.

Document 2

Source: Treaty of Greenville, August 1795.

A treaty of peace between the United States of America and the tribes of Indians called the Wyandots,
Delawares, Shawnees, Ottawas, Chippewas, Potawatomis, Miamis, Eel Rivers, Weas, Kickapoos,
Piankashaws, and Kaskaskias. . . .

Article 1st: Henceforth all hostilities shall cease; peace is hereby established, and shall be perpetual; and a
friendly intercourse shall take place between the said United States and Indian tribes. . . .

Article 4th: In consideration of the peace now established . . . the United States relinquish their claims to all
other Indian lands northward of the River Ohio, eastward of the Mississippi, and westward and southward of
the great lakes and the waters uniting them, according to the boundary line agreed on by the United States
and the King of Great Britain, in the treaty of peace made between them in the year 1783. . . .

Article 5th: To prevent any misunderstanding about the Indian lands relinquished by the United States . . . the
meaning of that relinquishment is this: the Indian tribes who have a right to those lands, are quietly to enjoy
them, hunting, planting, and dwelling thereon so long as they please, without any molestation from the
United States; but when those tribes or any of them shall be disposed to sell their lands, or any part of them,
they are to be sold only to the United States, and until such sale, the United States will protect all the said
Indian tribes in the quiet enjoyment of their lands against all citizens of the United States, and against all
other white persons who intrude upon the same. And the said Indian tribes again acknowledge themselves
and all their people to be under the protection of the said United States and no other power whatever.

-5- GO ON TO THE NEXT PAGE.

Downloaded by Loadster (kadenpizza66@gmail.com)


lOMoARcPSD|34158146

Document 3

Source: Letter from Thomas Jefferson to James Monroe (United States minister to France), September 1795.

Mr. Jay’s treaty has at length been made public. So general a burst of dissatisfaction never before appeared
against any transaction. Those who understand the particular articles of it, condemn these articles. Those who
do not understand them minutely, condemn it generally as wearing a hostile face to France. This last is the
most numerous class, comprehending the whole body of the people, who have taken a greater interest in this
transaction than they were ever known to do in any other. It has, in my opinion, completely demolished the
monarchical party here. The chamber of commerce in New York, against the body of the town, the merchants
in Philadelphia, against the body of their town, also, and our town of Alexandria have come forward in its
support.

Document 4

Source: James L. Cathcart, United States consul at Tripoli, letter to the Secretary of State, May 1800.

The only conclusion which can be drawn from the Bashaw’s [pasha of Tripoli, Yusuf Karamanli] proceedings
is that he wants a present, and if he does not get one, he will forge pretences to commit depredations on the
property of our fellow citizens; His letter to the President will be the means of keeping him quiet until he
receives an answer. . . . I therefore can see no alternative, but to station some of our Frigates in the
Mediterranean, otherwise we will be continually subject to the same insults which the
Imperials [Austria-Hungary], Danes, Swedes, and Ragusians [region in modern day Croatia] have already
suffered and will still continue to suffer, if they do not keep a sufficient Naval force in this Sea to protect their
trade.

-6- GO ON TO THE NEXT PAGE.

Downloaded by Loadster (kadenpizza66@gmail.com)


lOMoARcPSD|34158146

Document 5

Source: United States Foreign Trade, 1790–1812, from Historical Statistics of the United States, Colonial
Times to 1970.

-7- GO ON TO THE NEXT PAGE.

Downloaded by Loadster (kadenpizza66@gmail.com)


lOMoARcPSD|34158146

Document 6

Source: Secretary of the Treasury Alexander James Dallas, An Exposition of the Causes and Character of the
War between the United States and Great-Britain, 1815.

The United States had just recovered, under the auspices of their present constitution, from the debility which
their revolutionary struggle had produced, when the convulsive movements of France excited throughout the
civilized world the mingled sensations of hope and fear—of admiration and alarm. The interest which those
movements, would in themselves, have excited, was incalculably increased, however, as soon as Great Britain
became a party to the first memorable coalition against France, and assumed the character of a belligerent
power . . . the difference of the scene would no longer exempt the United States from the influence, and the
evils of the European conflict. On the one hand, their government was connected with France, by treaties of
alliance and commerce; and the services which that nation had rendered to the cause of American
independence. . . . On the other hand, Great Britain leaving the treaty of 1783 unexecuted, forcibly retained
the American posts upon the northern frontier; and, slighting every overture to place the diplomatic and
commercial relations of the two countries, upon a fair and friendly foundation, seemed to contemplate the
success of the American revolution, in a spirit of unextinguishable animosity.

Document 7

Source: Secretary of State John Quincy Adams, an address delivered at the request of the Committee of the
Citizens of Washington, on the occasion of reading the Declaration of Independence, July 4, 1821.

America, in the assembly of nations, since her admission among them, has invariably, though often
fruitlessly, held forth to them the hand of honest friendship, of equal freedom, of generous reciprocity. She
has uniformly spoken among them, though often to heedless and often to disdainful ears, the language of
equal liberty, of equal justice, and of equal rights. She has, in the lapse of nearly half a century, without a
single exception, respected the independence of other nations while asserting and maintaining her own. She
has abstained from interference in the concerns of others, even when the conflict has been for principles to
which she clings, as to the last vital drop that visits the heart. . . .

But she goes not abroad, in search of monsters to destroy. She is the well-wisher to the freedom and
independence of all. She is the champion and vindicator only of her own. . . .

She well knows that by once enlisting under other banners than her own, were they even the banners of
foreign Independence, she would involve herself, beyond the power of extrication, in all the wars of interest
and intrigue, of individual avarice, envy, and ambition, which assume the colors and usurp the standard of
freedom. The fundamental maxims of her policy would insensibly change from liberty to force. . . . She
might become the dictatress of the world. She would be no longer the ruler of her own spirit.

END OF DOCUMENTS FOR QUESTION 1

-8- GO ON TO THE NEXT PAGE.

Downloaded by Loadster (kadenpizza66@gmail.com)


lOMoARcPSD|34158146

Question 2, 3, or 4 (Long Essay)


Suggested writing time: 40 minutes

Directions: Answer Question 2 or Question 3 or Question 4.

In your response you should do the following.

• Respond to the prompt with a historically defensible thesis or claim that establishes a line of reasoning.
• Describe a broader historical context relevant to the prompt.
• Support an argument in response to the prompt using specific and relevant examples of evidence.
• Use historical reasoning (e.g., comparison, causation, continuity or change over time) to frame or structure an
argument that addresses the prompt.
• Use evidence to corroborate, qualify, or modify an argument that addresses the prompt.

2. Evaluate the extent to which trans-Atlantic exchanges affected colonial culture in British North America in the
period from 1700 to 1770.

3. Evaluate the extent to which immigration affected United States culture in the period from 1840 to 1898.

4. Evaluate the extent to which sustained economic growth affected United States culture in the period from 1940
to 1970.

WHEN YOU FINISH WRITING, CHECK YOUR WORK ON SECTION II IF TIME PERMITS.

-9- GO ON TO THE NEXT PAGE.

Downloaded by Loadster (kadenpizza66@gmail.com)


lOMoARcPSD|34158146

STOP

END OF EXAM
________________________________

THE FOLLOWING INSTRUCTIONS APPLY TO THE COVERS OF THE SECTION II BOOKLET.

• APPLY THE LABEL THAT CORRESPONDS TO THE LONG ESSAY QUESTION YOU
ANSWERED, AS REQUESTED ON THE FRONT COVER.
• MAKE SURE YOU HAVE COMPLETED THE IDENTIFICATION INFORMATION AS
REQUESTED ON THE FRONT AND BACK COVERS OF THE SECTION II BOOKLET.
• CHECK TO SEE THAT YOUR AP NUMBER LABEL APPEARS IN THE BOX ON THE
FRONT COVER.
• MAKE SURE YOU HAVE USED THE SAME SET OF AP NUMBER LABELS ON ALL
AP EXAMS YOU HAVE TAKEN THIS YEAR.

-10-

Downloaded by Loadster (kadenpizza66@gmail.com)


lOMoARcPSD|34158146

Student Answer Sheet for


the Multiple-Choice Section

Use this section to capture student responses. (Note that the following
answer sheet is a sample, and may differ from one used in an actual exam.)

Downloaded by Loadster (kadenpizza66@gmail.com)


lOMoARcPSD|34158146

Answer Key for AP United States History


Practice Exam, Section I

Question 1: A Question 29: D


Question 2: B Question 30: A
Question 3: B Question 31: C
Question 4: C Question 32: B
Question 5: C Question 33: A
Question 6: C Question 34: A
Question 7: A Question 35: D
Question 8: D Question 36: B
Question 9: C Question 37: A
Question 10: A Question 38: C
Question 11: D Question 39: C
Question 12: A Question 40: B
Question 13: C Question 41: D
Question 14: B Question 42: B
Question 15: C Question 43: D
Question 16: A Question 44: A
Question 17: B Question 45: C
Question 18: A Question 46: A
Question 19: C Question 47: B
Question 20: A Question 48: D
Question 21: C Question 49: C
Question 22: D Question 50: C
Question 23: B Question 51: B
Question 24: B Question 52: A
Question 25: D Question 53: B
Question 26: C Question 54: A
Question 27: A Question 55: B
Question 28: D

Downloaded by Loadster (kadenpizza66@gmail.com)


lOMoARcPSD|34158146

Free-Response Scoring Guidelines

The following contains the scoring guidelines


for the free-response questions in this exam.

Downloaded by Loadster (kadenpizza66@gmail.com)


lOMoARcPSD|34158146

AP® UNITED STATES HISTORY


2018 SCORING GUIDELINES

Short-Answer Question 1

“If American sectionalism entered a new phase in 1846, it was neither because North and South clashed for the
first time nor because the issue of slavery for the first time assumed importance. As early as the
Confederation, North and South had been at odds. . . . Once the government under the Constitution went into
effect, bitter sectional conflicts raged. . . . This sectional rivalry tended to become institutionalized in the
opposing [political] organizations. . . . No matter which region embraced nationalism and which particularism,
sectional conflict remained a recurrent phenomenon.”

David M. Potter, The Impending Crisis, 1848–1861, published in 1976

“The Civil War represented an utter and unique breakdown of the normal democratic political process. When
one section of the country refused to accept the decision of a presidential election, secession and the ensuing
war became the great exception to the American political tradition of compromise. The rending of the nation
was the one time that conflict seemed too irrepressible, too fundamental, to be contained within common
consensual boundaries. Because the war was such an anomaly, both participants and later historians have
been fascinated with its causes since the shooting started.”

Michael F. Holt, The Political Crisis of the 1850s, published in 1978

1. Using the excerpts above, answer (a), (b), and (c).

a) Briefly describe ONE major difference between Potter’s and Holt’s historical interpretations of the Civil War.
b) Briefly explain how ONE specific historical event or development during the period 1786–1861 that is not
explicitly mentioned in the excerpts could be used to support Potter’s interpretation.
c) Briefly explain how ONE specific historical event or development during the period 1786–1861 that is not
explicitly mentioned in the excerpts could be used to support Holt’s interpretation.

Scoring Guide

0–3 points

Score 3
Response accomplishes all three tasks set by the question.

Score 2
Response accomplishes two of the tasks set by the question.

Score 1
Response accomplishes one of the tasks set by the question.

Score 0
Response accomplishes none of the tasks set by the question.

Score NR
Is completely blank

© 2018 The College Board.


Visit the College Board on the Web: www.collegeboard.org.

Downloaded by Loadster (kadenpizza66@gmail.com)


lOMoARcPSD|34158146

AP® UNITED STATES HISTORY


2018 SCORING GUIDELINES

Short-Answer Question 1 (continued)

Question-Specific Scoring Guide


• ONE point for describing one major difference between Potter’s and Holt’s historical interpretations
of the Civil War
• ONE point for explaining how one specific historical event or development during the period
1786–1861 that is not explicitly mentioned in the excerpts could be used to support Potter’s
interpretation
• ONE point for explaining how specific historical event or development during the period 1786–1861
that is not explicitly mentioned in the excerpts could be used to support Holt’s interpretation

Scoring Notes

Introductory notes:
• Each point is earned independently.
• Accuracy: These rubrics require that students demonstrate historically defensible content knowledge.
Given the timed nature of the exam, responses may contain errors that do not detract from their overall
quality, as long as the historical content used to advance the argument is accurate.
• Clarity: Exam responses should be considered first drafts and thus may contain grammatical errors.
Those errors will not be counted against a student unless they obscure the successful demonstration
of the content knowledge, skills, and practices described below.

Examples of responses to (a) that would earn credit:


• Potter argues that the Civil War was a continuation of the sectionalism and conflict that
characterized United States politics since the creation of the nation, and, thus, it was not a change
or break from the past.
• In contrast, Holt contends that the Civil War represented an unprecedented breakdown of United
States politics, a sharp contrast to the general consensus in the political system in the period prior
to the outbreak of the war.
• Credited responses must explicitly address the substance of both excerpts.

Examples of responses to (b) that would earn credit:


• Northern and Mid-Atlantic states abolition of slavery or gradual emancipation
• Northwest Ordinance outlaws slavery above the Ohio River in Northwest Territory
• Missouri Compromise as evidence of tensions and sectionalism
• Development of political parties with regional loyalties
• Crises over tariffs and nullification (Tariff of Abominations; Kentucky and Virginia Resolutions)
• Impasses over annexation/acquisition of new territory
• Breakdown of series of compromises in the 1850s (Compromise of 1850, Kansas-Nebraska Act,
Bleeding Kansas)
• Growing economic division between the North and South (e.g., Industrialization, Transportation,
and Communication)
• The Hartford Convention as an example of Northern sectional interest causing conflict with the South.
• The enactment of the “Gag Rule” prohibiting the discussion of slavery in Congress

© 2018 The College Board.


Visit the College Board on the Web: www.collegeboard.org.

Downloaded by Loadster (kadenpizza66@gmail.com)


lOMoARcPSD|34158146

AP® UNITED STATES HISTORY


2018 SCORING GUIDELINES

Short-Answer Question 1 (continued)

Examples of responses to (c) that would earn credit:


NOTE: Any conflicts would have to have occurred after 1850 to earn credit (the Mexican-American War
is not creditable).
• Election of Lincoln in 1860 (note, the excerpt does not include reference to any specific election
or candidate)
• Compromises in Constitution
• Opposing sense of nationalism within the two separate regions
• Series of compromises over addition of new territories
• Bleeding Kansas and Kansas-Nebraska Act
• Cotton economy bound together economic interests of two regions
• Fugitive Slave Act
• Uncle Tom’s Cabin
• Dred Scott v. Sandford

NOTE: It would be acceptable for test-takers to use some of the same examples to respond to both (b) and (c),
as long as they explain how the example supported both interpretations.

© 2018 The College Board.


Visit the College Board on the Web: www.collegeboard.org.

Downloaded by Loadster (kadenpizza66@gmail.com)


lOMoARcPSD|34158146

AP® UNITED STATES HISTORY


2018 SCORING GUIDELINES

Short-Answer Question 2

1. Using the 1928 image above, answer (a), (b), and (c).
a) Briefly describe ONE perspective about women’s roles during the 1920s expressed through the image.
b) Briefly explain ONE specific historical development or circumstance from 1900 to 1929 that led to changes
in women’s roles during the 1920s such as that depicted in the image.
c) Briefly explain ONE specific historical development in business OR culture that led to the creation and use
of images such as the one above during the 1920s.

© 2018 The College Board.


Visit the College Board on the Web: www.collegeboard.org.

Downloaded by Loadster (kadenpizza66@gmail.com)


lOMoARcPSD|34158146

AP® UNITED STATES HISTORY


2018 SCORING GUIDELINES

Short-Answer Question 2 (continued)

Scoring Guide

0–3 points

Score 3
Response accomplishes all three tasks set by the question.

Score 2
Response accomplishes two of the tasks set by the question.

Score 1
Response accomplishes one of the tasks set by the question.

Score 0
Response accomplishes none of the tasks set by the question.

Score NR
Is completely blank

Question-Specific Scoring Guide

• ONE point for describing one perspective about women’s roles during the 1920s expressed through
the image
• ONE point for explaining one specific historical development or circumstance from 1900 to 1929
that led to changes in women’s roles during the 1920s such as that depicted in the image
• ONE point for explaining one specific historical development in business OR culture that led
to the creation and use of images such as the one above during the 1920s

Scoring Notes

Introductory notes:
• Each point is earned independently.
• Accuracy: These rubrics require that students demonstrate historically defensible content knowledge.
Given the timed nature of the exam, responses may contain errors that do not detract from their overall
quality, as long as the historical content used to advance the argument is accurate.
• Clarity: Exam responses should be considered first drafts and thus may contain grammatical errors.
Those errors will not be counted against a student unless they obscure the successful demonstration
of the content knowledge, skills, and practices described below.

Examples of responses to (a) that would earn credit:


• Women of the 1920s were able to take more active roles in society than in past decades.
• Women of the 1920s were able to act more freely than in past decades and pursue adventures
(i.e., through driving independently, making their own choices about purchases, and being a flapper).
• Women of the 1920s advocated the Equal Rights Amendment.
• Women of the 1920s made strides forward, but did not achieve full equality. Traditional gender
roles lingered.

© 2018 The College Board.


Visit the College Board on the Web: www.collegeboard.org.

Downloaded by Loadster (kadenpizza66@gmail.com)


lOMoARcPSD|34158146

AP® UNITED STATES HISTORY


2018 SCORING GUIDELINES

Short-Answer Question 2 (continued)

Examples of responses to (b) that would earn credit:


• Sex discrimination eliminated in voting; Nineteenth Amendment (1920)
• Women had played an active, supportive role during the First World War and involvement in more
public and political roles continued
• Mass media influenced culture and women were as likely to become highlighted in advertising
as were men
• More economic opportunities for women; access to jobs in offices, sales, factories, and elsewhere
• Rise of flappers and the activities associated with flappers in the 1920s
• Women played active roles in social reform movements of Progressive Era

Examples of responses to (c) that would earn credit:


• New technologies (e.g., print magazines, moving pictures, and the radio in the 1920s) allowed more
Americans to get information more quickly.
• Newly developed forms of mass media (e.g., the radio and advertising such as roadside billboards)
influenced a developing consumer culture.
• The advertising industry sought to cultivate new markets (i.e., female drivers, independent women,
flappers and young women).
• Economic growth allowed improved quality of life, encouraging people to spend disposable income
on new consumer goods.
• The combination of mass production and increasing mass marketing toward women expanded
opportunities for consumption and economic decision making.
• Availability of credit and peoples’ willingness to buy on the installment plan expanded access
to consumer goods.

NOTE: some examples could be used to support either (b) or (c). Responses should clearly indicate the connection
between the historical development explained and the point that it is supporting.

© 2018 The College Board.


Visit the College Board on the Web: www.collegeboard.org.

Downloaded by Loadster (kadenpizza66@gmail.com)


lOMoARcPSD|34158146

AP® UNITED STATES HISTORY


2018 SCORING GUIDELINES

Short-Answer Question 3

Answer (a), (b), and (c), confining your response to the period 1700 to 1776.
a) Briefly describe ONE specific historical difference in North America between the First Great Awakening
and the Enlightenment.
b) Briefly describe ONE specific historical similarity in North America between the First Great Awakening
and the Enlightenment.
c) Briefly explain ONE specific historical effect in North America of either the Great Awakening or
the Enlightenment.

Scoring Guide

0–3 points

Score 3
Response accomplishes all three tasks set by the question.

Score 2
Response accomplishes two of the tasks set by the question.

Score 1
Response accomplishes one of the tasks set by the question.

Score 0
Response accomplishes none of the tasks set by the question.

Score NR
No response. Response is completely blank.

Question-Specific Scoring Guide

• ONE point for describing one specific historical difference in North America between the First Great
Awakening and the Enlightenment
• ONE point for describing ONE specific historical similarity in North America between the First Great
Awakening and the Enlightenment
• ONE point for explaining one specific historical effect in North America of either the Great Awakening
or the Enlightenment

© 2018 The College Board.


Visit the College Board on the Web: www.collegeboard.org.

Downloaded by Loadster (kadenpizza66@gmail.com)


lOMoARcPSD|34158146

AP® UNITED STATES HISTORY


2018 SCORING GUIDELINES

Short-Answer Question 3 (continued)

Scoring Notes

Introductory notes:
• Each point is earned independently.
• Accuracy: These rubrics require that students demonstrate historically defensible content knowledge.
Given the timed nature of the exam, responses may contain errors that do not detract from their overall
quality, as long as the historical content used to advance the argument is accurate.
• Clarity: Exam responses should be considered first drafts and thus may contain grammatical errors.
Those errors will not be counted against a student unless they obscure the successful demonstration
of the content knowledge, skills, and practices described below.

Examples of responses to (a) that would credit:


• First Great Awakening (or Great Awakening) about religion, personal revelation and need for salvation,
revivalism, pietism, emphasis on emotion, enthusiasm, and zealousness
• Enlightenment about rationalism, science, philosophy, democratic ideas, deism, and the secularization
of society
• Great Awakening stressed emotion as way to understand the world and the individual’s place in it,
while the Enlightenment stressed use of reason

NOTE: Credited responses must explicitly address both the Enlightenment and the First Great Awakening.

Examples of responses to (b) that would credit:


• Both sets of ideas originated in Europe and moved to North America and became mass movements.
• Both movements undermined allegiance or resulted in challenges to traditional authority, whether in
the form of the established churches and minsters or centralized political authority (British Crown).
• Both movements placed emphasis on the importance of the individual, personal experience as the
source of knowledge or revelation/salvation.
• Both movements promoted literacy, education, democracy.

Examples of responses to (c) that would earn credit:


• Great Awakening contributed to challenges to authority through emphasis on individual; many
churches split into New Lights and Old Lights; new denominations formed
• Enlightenment caused a questioning of authority and concepts of natural rights reinforced idea
of human ability to govern themselves; contributed to intellectual origins of American Revolution,
particularly evident in documents such as Thomas Jefferson’s Declaration of Independence;
contributed to the rise of universities

© 2018 The College Board.


Visit the College Board on the Web: www.collegeboard.org.

Downloaded by Loadster (kadenpizza66@gmail.com)


lOMoARcPSD|34158146

AP® UNITED STATES HISTORY


2018 SCORING GUIDELINES

Short-Answer Question 4

Answer (a), (b), and (c).


a) Briefly describe ONE specific historical similarity between the United States reasons for involvement
in the Korean War and its reasons for involvement in the Vietnam War.
b) Briefly describe ONE specific historical difference between the United States experience in the Korean War
and in the Vietnam War.
c) Briefly explain ONE specific historical effect of either the Korean War or the Vietnam War on
United States society.

Scoring Guide

0–3 points

Score 3
Response accomplishes all three tasks set by the question.

Score 2
Response accomplishes two of the tasks set by the question.

Score 1
Response accomplishes one of the tasks set by the question.

Score 0
Response accomplishes none of the tasks set by the question.

Score NR
Is completely blank

Question-Specific Scoring Guide

• ONE point for describing one specific historical similarity between the United States reasons
for involvement in the Korean War and its reasons for involvement in the Vietnam War
• ONE point for describing ONE specific historical difference between the United States experience
in the Korean War and in the Vietnam War
• ONE point for explaining specific historical effect of either the Korean War or the Vietnam War
on United States society

Scoring Notes

Introductory notes:
• Each point is earned independently.
• Accuracy: These rubrics require that students demonstrate historically defensible content knowledge.
Given the timed nature of the exam, responses may contain errors that do not detract from their overall
quality, as long as the historical content used to advance the argument is accurate.
• Clarity: Exam responses should be considered first drafts and thus may contain grammatical errors.
Those errors will not be counted against a student unless they obscure the successful demonstration
of the content knowledge, skills, and practices described below.

© 2018 The College Board.


Visit the College Board on the Web: www.collegeboard.org.

Downloaded by Loadster (kadenpizza66@gmail.com)


lOMoARcPSD|34158146

AP® UNITED STATES HISTORY


2018 SCORING GUIDELINES

Short-Answer Question 4 (continued)

Examples of responses to (a) that would earn credit:

• The United States believed it had to contain the spread and influence of communism around the world.
• Supporting democratic governments and leaders/supporting democracy around the world as vital
to United States interests.
• The United States had an interest in maintaining free markets and capitalism around the world.
• The United States played a role in managing unresolved territorial (Soviet Union and China,
interest in Korea) and colonial claims (France’s interest in Vietnam/Indochina) from the immediate
post-Second World War era.

Examples of responses to (b) that would earn credit:

• North Korea’s invasion of South Korea prompted a United States and United Nations action
to drive the North Koreans out, while in Vietnam the United States intervened because of the defeat
of the French and the division of Vietnam decided at the peace conference.
• United States intervened in South Vietnam to build a government that would serve as an alternative
to the Communist North Vietnam; US provided funds and support for the South Vietnamese regime
against North Vietnam and the Viet Cong.
• Soldiers’ wartime experience, especially of guerilla warfare, differed in Vietnam.
• United States experience in Korea validated, to some extent, the concept of containing communism,
while the experience and United States loss in Vietnam ultimately undermined the concept
of containment and preventing the domino effect.
• Domestic differences may include the more dramatic opposition to involvement in Vietnam as well as
the diversion of energy and funds from domestic programs such as the Great Society.
• NOTE: Credited responses must address both the Korean War and the Vietnam War, but can imply the
other with comparative qualifiers (such as “Vietnam was more divisive” thus implying “than Korea”)
and describe more than vague generalizations.

Examples of responses to (c) that would earn credit:


NOTE: Response may use the same example in c as b, BUT earns the point only with additional
development of the example.
• Korean War
o Confirmed domestic anxieties about the threat of communism and the Soviet Union and China
o Led to a sustained period of Cold War tensions
o Extended use of the Selective Service System (the draft) from 1950 through the 1960s and
the service of millions in the Armed forces at home and abroad in this period
• Vietnam War
o Anti-war protest movement in the United States developed in response to United States
involvement in Vietnam
o Young people protested the military draft
o Growing divide in the United States between traditionalists/conservatives/older Americans
and leftists/radicals/younger Americans
o Led to neglect of domestic programs such as Great Society, the downfall of the Lyndon
Johnson presidency in 1968, and public distrust of government (“credibility gap”)

© 2018 The College Board.


Visit the College Board on the Web: www.collegeboard.org.

Downloaded by Loadster (kadenpizza66@gmail.com)


lOMoARcPSD|34158146

AP® UNITED STATES HISTORY


2018 SCORING GUIDELINES

Question 1 — Document-Based Question

Evaluate the extent of change in United States foreign policy in the period 1783 to 1828.

Maximum Possible Points: 7

Points Rubric Notes


Responds to the prompt with a The thesis must make a historically defensible
historically defensible thesis/claim claim that establishes a line of reasoning about
that establishes a line of reasoning change in United States foreign policy in the
(1 point) period 1783 to 1828.
A: Thesis/Claim (0–1)

To earn this point, the thesis must Examples that earn this point include:
make a claim that responds to the • “Although the United States tried its best to
prompt rather than restating or adhere to Washington’s pleas for neutrality,
rephrasing the prompt. The thesis the events on the world stage, from British
must consist of one or more sentences impressment to the wars in Europe, forced the
located in one place, either in the United States to occasionally engage in foreign
introduction or the conclusion. affairs that even led to war in the 1810s.”
• “Between 1783 and 1828 the United States
shifted away from isolation toward acting more
like a world power.”

Describes a broader historical To earn the point, the response must accurately
context relevant to the prompt describe a context relevant to change in United
(1 point) States foreign policy in the period 1783 to 1828.

To earn this point, the response must Examples of context might include the following,
relate the topic of the prompt to with appropriate elaboration.
B: Contextualization

broader historical events, • The American Revolution; US alliance with


developments, or processes that occur France
• Mercantilism; commercial dependence on
(0–1)

before, during, or continue after the


time frame of the question. This point Great Britain
is not awarded for merely a phrase • The French Revolution (1789); Haitian
or reference. Revolution (1794)
• French Revolutionary and Napoleonic Wars
(1794–1815)
• The First Party System
• Settler colonialism; American Indian
diplomacy, land cessions, and resistance
• Latin American independence movements

© 2018 The College Board.


Visit the College Board on the Web: www.collegeboard.org.

Downloaded by Loadster (kadenpizza66@gmail.com)


lOMoARcPSD|34158146

AP® UNITED STATES HISTORY


2018 SCORING GUIDELINES

Question 1 — Document-Based Question (continued)

Points Rubric Notes


Document Content: Uses the content See document summaries page for details.
of at least three documents to address • Doc 1: President George Washington,
the topic of the prompt (1 point) Neutrality Proclamation, 1793
To earn one point, the response must • Doc 2: Treaty of Greenville, 1795
accurately describe—rather than simply • Doc 3: Letter from Thomas Jefferson to
quote—the content from at least three of James Monroe, 1795
the documents. • Doc 4: James L. Cathcart, United States
OR consul at Tripoli, letter to the Secretary of
Supports an argument in response to State, 1800
the prompt using at least six • Doc 5: Graph, United States Foreign Trade,
documents (2 points) 1790–1812
To earn two points, the response must • Doc 6: Secretary of the Treasury Alexander
accurately describe—rather than simply James Dallas, An Exposition of the Causes
quote—the content from at least six and Character of the War between the United
documents. In addition, the response States and Great-Britain, 1815.
must use the content from the • Doc 7: Secretary of State John Quincy
documents to support an argument in Adams, address delivered on the occasion
response to the prompt. of reading the Declaration of Independence,
C: Evidence (0–3)

July 4, 1821
Evidence Beyond the Documents: Evidence used might include the following, with
Uses at least one additional piece of appropriate elaboration:
specific historical evidence (beyond • Adam Smith, Wealth of Nations (1776)
that found in the documents) relevant • Battle of Fallen Timbers (1795)
to an argument about the prompt • XYZ Affair (1798)
(1 point) • Quasi-war with France (1798–1800)
• Alien and Sedition Acts (1798)
To earn this point, the evidence must be • Kentucky and Virginia Resolutions (1798)
described and must be more than a • Revolution of 1800
phrase or reference. This additional • Embargo Act of 1807
piece of evidence must be different from • Tenskwatawa (The Prophet)
the evidence used to earn the point for • Tecumseh
contextualization. • Battle of Tippecanoe (1811)
• Impressment
• War of 1812
• Treaty of Ghent (1814)
• Monroe Doctrine (1823)

© 2018 The College Board.


Visit the College Board on the Web: www.collegeboard.org.

Downloaded by Loadster (kadenpizza66@gmail.com)


lOMoARcPSD|34158146

AP® UNITED STATES HISTORY


2018 SCORING GUIDELINES

Question 1 — Document-Based Question (continued)

Points Rubric Notes


Sourcing: For at least three See document summaries page for examples of
documents, explains how or why the possible explanations of the relevance of
document’s point of view, purpose, sourcing.
historical situation, and/or audience is
relevant to an argument (1 point)

To earn this point, the evidence must


explain how or why—rather than simply
identifying—the document’s point of
view, purpose, historical situation, or
audience is relevant to an argument
about the prompt for each of the three
documents sourced.
Complexity: Demonstrates a complex Examples of demonstrating a complex
understanding of the historical understanding might include:
D: Analysis and Reasoning (0–2)

development that is the focus of the • Explaining a nuance by exploring different


prompt, using evidence to corroborate, ways in which United States foreign policy
qualify, or modify an argument that changed as a result of overseas conflicts and
addresses the question (1 point) conflicts in North America.
• Explaining similarities and differences in the
A response may demonstrate a complex United States reaction to conflicts in different
understanding in a variety of ways, parts of the world.
such as the following. • Explaining connections to United States
• Explaining a nuance of an issue by foreign policy in other time periods such as
analyzing multiple variables imperialism in the late nineteenth century.
• Explaining both similarities and • Confirming the validity of the response’s
differences, or explaining both argument by explaining how different
continuity and change, or explaining documents corroborate the argument about
multiple causes, or explaining both United States foreign policy in spite of
causes and effects differing perspectives among Federalist and
• Explaining relevant and insightful Republican authors.
connections within and across periods • Qualifying or modifying the argument by
• Confirming the validity of an considering the differences in how the
argument by corroborating multiple United States responded militarily to Native
perspectives across themes American politics as opposed to seeking
• Qualifying or modifying an argument neutrality in European politics.
by considering diverse or alternative
views or evidence
This understanding must be part of the
argument, not merely a phrase or
reference.
If response is completely blank, enter -- for all four score categories A, B, C, and D

© 2018 The College Board.


Visit the College Board on the Web: www.collegeboard.org.

Downloaded by Loadster (kadenpizza66@gmail.com)


lOMoARcPSD|34158146

AP® UNITED STATES HISTORY


2018 SCORING GUIDELINES

Question 1 — Document-Based Question (continued)

Document Summaries and Possible Sourcing

Document Summary of Content Response explains the relevance of point of view,


purpose, situation, and/or audience by elaborating
on examples such as the following.
1. Washington, • Declares United States neutrality • After independence, Americans worried that the
Neutrality in current war between France United States was too weak and not prepared for
Proclamation, and several major European military conflict with European powers. (situation)
1793 powers • Washington’s official statement of government
policy did not entirely reflect the public sentiment of
Federalists who sympathized with England and
Republicans who sympathized with France. (point
of view)
2. Treaty of • Establishes peace between US • White settler migration into the sovereign
Greenville, and a number of American Indian homelands of American Indians provoked conflicts
1795 nations with Indian groups and calls for federal military
• United States gives up claim to action. (situation)
specified American Indian lands • The treaty’s terms for American Indian possession
and sale of land sought to project the authority of
• Allows American Indians to sell
the new federal government in diplomacy with
lands to US
Native Americans. (purpose)
• Promises US protection of
• United States officials negotiating the treaty sought
American Indians on lands to establish a paternal relationship between the
federal government and American Indians. (point of
view)
3. Letter from • Communicates popular • Democratic Republicans, who are loyal to France
Thomas opposition to the Jay Treaty express joy in the public’s distaste of Jay’s treaty,
Jefferson to • Suggests the treaty has created a which was crafted by a Federalist (point of view)
James Monroe, political divide within the United • The treaty, which was designed to halt British
1795 States economic interference on the high seas, failed to
prevent the impressment of American sailors,
making it useless (situation)
4. James • Cites threat that leader of Tripoli • This occurs due to the Barbary Pirates not
Cathcart, US poses to US ships respecting American neutrality and attacking naval
consul at • Proposes using US naval forces vessels-thus the nation must use military force
Tripoli, letter to in the Mediterranean Sea to (purpose)
the Secretary protect US trade • The end of British protection of the former colonies
of State, 1800 meant that Americans would now have to deal with
the pirates who had been previously been paid
ransoms by the British Empire (situation)
5. Graph, • Graph shows general increase in • Dramatic drop in 1808 due to Jefferson’s Embargo
United States US foreign trade from 1790 to Act as a means of averting military action
Foreign Trade, 1807, sharp drop in 1808, and (situation)
1790–1812 then recovery by 1812 but still
not at previous peak

© 2018 The College Board.


Visit the College Board on the Web: www.collegeboard.org.

Downloaded by Loadster (kadenpizza66@gmail.com)


lOMoARcPSD|34158146

AP® UNITED STATES HISTORY


2018 SCORING GUIDELINES

Question 1 — Document-Based Question (continued)

Document Summary of Content Response explains the relevance of point of view,


purpose, situation, and/or audience by elaborating
on examples such as the following.
6. Dallas, • Seeks to explain reason for US • Dallas argued that the US trade with France made
Exposition of participation in War of 1812 the war inevitable and given his role as secretary of
the War against Great Britain the treasury, his economic analysis is accurate
between the • Cites US connections (point of view)
United States with France • Dallas argued that America was tied to France
and Great- • Cites British retention of North through their alliance during the Revolution and
Britain, 1815 American forts and still tied to Britain because their former parent
unwillingness to normalize country failed to follow through the peace treaty of
relations with US 1783. Thus, Madison had no choice but to declare
war when things with Britain (impressment) got out
of hand and US moved away from its neutral
standing. (purpose)
7. Secretary of • Argues US wants to be friendly • This sentiment is expressed during the Era of Good
State John with other countries but Feelings, a period of national unity in the aftermath
Quincy Adams, abstained from involving itself of the War of 1812, which saw political disputes
address, July with foreign concerns between Federalists, who evaporated as a political
4, 1821 • Asserts that the US supports party, and Democratic-Republicans end (point of
freedom and independence but view)
does not seek to impose its own • This address marked the new power of the US on
values abroad the emerging global stage while still maintaining
• Seeking to impose freedoms the idea of isolationism. (purpose)
abroad would change the US

© 2018 The College Board.


Visit the College Board on the Web: www.collegeboard.org.

Downloaded by Loadster (kadenpizza66@gmail.com)


lOMoARcPSD|34158146

AP® UNITED STATES HISTORY


2018 SCORING GUIDELINES

Question 1 — Document-Based Question (continued)

Scoring Notes

Introductory notes:
• Except where otherwise noted, each point of these rubrics is earned independently, e.g., a student
could earn a point for evidence without earning a point for thesis/claim.
• Accuracy: The components of these rubrics require that students demonstrate historically defensible
content knowledge. Given the timed nature of the exam, essays may contain errors that do not detract
from their overall quality, as long as the historical content used to advance the argument is accurate.
• Clarity: Exam essays should be considered first drafts and thus may contain grammatical errors.
Those errors will not be counted against a student unless they obscure the successful demonstration
of the content knowledge, skills, and practices described below.

Note: Student samples are quoted verbatim and may contain grammatical errors.

A. Thesis/Claim (0–1 points)

Responses earn one point by responding to the prompt with a historically defensible thesis that establishes
a line of reasoning about the topic. To earn this point, the thesis must make a claim that responds to the
prompt rather than simply restating or rephrasing the prompt. The thesis must suggest at least one main line
of argument development or establish the analytic categories of the argument.

The thesis must consist of one or more sentences located in one place, either in the introduction or
the conclusion.

Examples of acceptable theses


• “America’s foreign policy from 1783 to 1828 faced a double edged sword. On one hand, they tried
to stay neutral while being lured into the never ending conflict between Britain and France. On the
other hand, they sought to expand their trade and land while dealing with hostile Native Americans.”
(This example shows a complex and historically defensible line of argument).
• “The numerous restrictions from Great Britain, combined with the constant state of conflict from
the colonial times, turned the former colonies off from international conflict and caused foreign policy
from 1783-1828 to be rooted on neutrality.” (This example suggests a historically defensible line of
argument development.)
• “The United States sought to be neutral and confront domestic issues, the nation became increasingly
involved as it became more dependent on international trade and more intertwined in the tension
between Great Britain and France.” (This example establishes the analytic categories for the argument.)

Example of unacceptable thesis


• “United States foreign policy developed around the idea of Manifest Destiny.” (This example is too
generic and not specific to the time period.)
• “America’s foreign policy changed a lot because we were tired of Britain messing in our lives.”
(This response largely repeats the prompt without adding historical material particular to the prompt.)

© 2018 The College Board.


Visit the College Board on the Web: www.collegeboard.org.

Downloaded by Loadster (kadenpizza66@gmail.com)


lOMoARcPSD|34158146

AP® UNITED STATES HISTORY


2018 SCORING GUIDELINES

Question 1 — Document-Based Question (continued)

B. Contextualization (0–1 points)

Responses earn one point by describing a broader historical context relevant to the topic of the prompt.
To earn this point, the response must accurately and explicitly connect the context of the prompt to broader
historical events, developments, or processes that occurred before, during, or continued after the time frame
of the question. This point is not awarded for merely a phrase or reference.

To earn the point, the response must accurately describe a context relevant to change in United States foreign
policy in the period 1783 to 1828.

Examples of context might include:


• The American Revolution; alliance with France
• Mercantilism; commercial dependence on Great Britain
• The French Revolution (1789); Haitian Revolution (1794)
• French Revolutionary and Napoleonic Wars (1794–1815)
• The First Party System
• Settler colonialism; American Indian land cessions and resistance
• Latin American independence movements
• The Critical Period (Articles of Confederation, Constitutional Convention)

Example of acceptable contextualization:


• “Globalization has been a part of the American life before the true foundations of the nation were
formed. Through the Columbian Exchange, the trans-Atlantic exchanges, and the colonies relationship
with the mother country, Great Britain, the United States was destined to be internationally involved.”
(This response describes commercial dependence which is broader historical context relevant to the topic
of the prompt.)
• “The outbreak of the French Revolution in 1789 and subsequent warfare between Great Britain and
France created opposing views about how the United States should align itself in foreign affairs.”
(This response describes one broader historical context relevant to the topic of the prompt.)

Example of unacceptable contextualization:


• “The American Revolution left Americans prideful of their new nation but it gave them too much
responsibility.” (This example is an attempt at contextualization. However, it is not specific enough in
describing the actual consequences of the American Revolution and what responsibilities emerged because
of victory. To earn credit for contextualization, this example would need to describe how the context offered
is relevant to the topic.)
• “The volatility of foreign policy is similar to the rights of African Americans throughout our history.
An example of this is how even with slavery abolished in the North, the Dred Scott case essentially
returned it to legality with fugitive slave laws to support it.” (This example in itself would not earn credit
for contextualization because, even though it may be historically accurate, it is not self-evidently relevant to
the topic. To earn credit for contextualization, this example would need to describe how the context offered
is relevant to the topic.)

© 2018 The College Board.


Visit the College Board on the Web: www.collegeboard.org.

Downloaded by Loadster (kadenpizza66@gmail.com)


lOMoARcPSD|34158146

AP® UNITED STATES HISTORY


2018 SCORING GUIDELINES

Question 1 — Document-Based Question (continued)

C. Evidence (0–3 points)

Evidence from the Documents


In order to earn one point for using evidence from the documents, the response must address the topic
of the prompt by using at least three documents. To earn one point for evidence from the documents,
the response must accurately describe—rather than simply quote or paraphrase—content from at least three
of the documents to address the topic of the prompt.

Example of describing the content of a document:


• “In Document 2, the United States wrote a treaty with the Native Americans to stop all the fighting.”
(This example describes evidence from a document relevant to the topic, so it contributes toward the first
evidence point, but it does not use that evidence to support an argument about change in United States
foreign policy, so it does not contribute toward the second point.)

Example of unacceptably describing the content of a document:


• “In Document 2, neutrality is taken but with regard to internal conflicts; they returned American
Indians’ rightful land to them and promised no further conflicts amongst them.” (This example does
not contribute toward credit for describing evidence from the documents relevant to the topic because it
misinterprets the document.)

OR

In order to earn two points for using evidence from the documents, the response must support an argument
in response to the prompt by accurately using the content of at least six documents. To earn two points,
responses must accurately describe the document’s content; they cannot earn a point by merely quoting
or paraphrasing the documents with no connection to the topic of the prompt.

Example of supporting an argument using the content of a document:


• “Document 5 shows that at the turn of the 19th century, trade with the foreign powers of the world
greatly increased (until Jefferson initiated the Embargo) either due to the lowering of tariffs, or the
expansion of international trade networks. Exports and imports both rose dramatically, meaning some
stability in foreign policy regarding trade.” (This example connects Document 5 and trade to foreign
policy and therefore contributes toward two points for using the documents to support an argument.)

Example of unacceptably supporting an argument using the content of a document:


• “In 1808 trade imports and especially exports took a steep drop losing the country millions of dollars
(Document 5).” (This example describes evidence from a document relevant to the topic, so it contributes
toward the first evidence point, but it does not use that evidence to support an argument about change in
United States foreign policy, so it does not contribute toward the second point.)

© 2018 The College Board.


Visit the College Board on the Web: www.collegeboard.org.

Downloaded by Loadster (kadenpizza66@gmail.com)


lOMoARcPSD|34158146

AP® UNITED STATES HISTORY


2018 SCORING GUIDELINES

Question 1 — Document-Based Question (continued)

Evidence Beyond the Documents

In order to earn one point for evidence beyond the documents, the response must use at least one additional
piece of specific historical evidence (beyond that found in the documents) relevant to an argument that
addresses the topic. To earn this point, the evidence must be described and must be more than a phrase
or reference.

This additional piece of evidence must be different from the evidence used to earn the point for
contextualization. Typically, statements credited as contextualization will be more general statements
that place an argument or a significant portion of it in a broader context. Statements credited as supporting
evidence beyond the documents will typically be more specific details that function as support for a
particular point made in an argument, analogous to the function of evidence drawn from the documents.

Evidence used might include the following, with appropriate elaboration:


• Adam Smith, Wealth of Nations (1776)
• Pinckney’s Treaty (1795)
• Western Confederacy; Chief Little Turtle (Miami Indians)
• Battle of Fallen Timbers (1795)
• XYZ Affair (1798)
• Quasi-war with France (1798-1800)
• Alien and Sedition Acts (1798)
• Kentucky and Virginia Resolutions (1798)
• Revolution of 1800
• Louisiana Purchase (1803)
• Embargo Act of 1807
• Non-Intercourse Act (1809)
• Macon’s Bill No. 2 (1810)
• Tenskwatawa (The Prophet)
• Tecumseh
• Battle of Tippecanoe (1811)
• Impressment
• War of 1812
• Treaty of Ghent (1814)
• Monroe Doctrine (1823)
• Rush-Bagot Treaty (1817)
• Convention/Treaty of 1818
• Adams-Onís Treaty (1819); acquisition of Florida

© 2018 The College Board.


Visit the College Board on the Web: www.collegeboard.org.

Downloaded by Loadster (kadenpizza66@gmail.com)


lOMoARcPSD|34158146

AP® UNITED STATES HISTORY


2018 SCORING GUIDELINES

Question 1 — Document-Based Question (continued)

Examples of acceptable use of an additional piece of specific historical evidence:


• “The War of 1812 between Tecumseh’s army of Native Americans and Americans accelerated the
clashes between the two groups. However, the defeat of Tecumseh led to the downfall of peaceful
coexistence between Natives and citizens and eventually leads to removal treaties that prompted the
Trail of Tears in the future under Andrew Jackson’s presidency.” (This example uses a particular piece
of evidence beyond the documents to make a connection to a larger argument.)
• “However, the U.S. eventually was forced to engage with foreign powers with the War of 1812.
The conflict was due to British ships impressing American sailors. The British claimed to be retaking
back their sailors. Jefferson attempted to limit impressment with the Embargo Act of 1807 and its
effects can be seen in Document 5 where there is a sharp decrease in imports during this year.”
(This example uses two particular pieces of evidence beyond the documents to make a connection to
a larger argument.)

Example of unacceptable use of an additional piece of specific historical evidence:


• “James Monroe is also significant because he wrote the Monroe Doctrine which called for more
European influence in the western hemispheres with hopes to make the U.S. the hegemony in that
half of the world.” (This example would not earn credit for evidence beyond the documents because it is
historically inaccurate.)

D. Analysis and Reasoning (0–2 points)

Document Sourcing
For at least three documents, the response explains how or why the document’s point of view, purpose,
historical situation, and/or audience is relevant to an argument that addresses the prompt. To earn this point,
the response must explain how or why—rather than simply identifying—the document’s point of view,
purpose, historical situation, or audience is relevant to an argument addressing the prompt for each of the
three documents sourced.

Example of an acceptable explanation of the relevance of the document’s point of view:


• “Jefferson, a Democratic-Republican, writes in Document 3 to inform Monroe (also a
Democratic-Republican) about the unpopularity of Jay’s treaty in the United States. With Monroe
stationed in France, he would not have been aware of the public’s disapproval and that he could
hopefully change things overseas.” (This example contributes toward a point for document sourcing
because it connects Jefferson and Monroe’s perspective on foreign affairs.)

Example of an unacceptable explanation of the relevance of the document’s point of view:


• “The point of view of this document (1) is of George Washington. He was a great general and
president so it could be said that he really did know best for his country, and that his words are true.”
(This example is a simplistic understanding of point of view since it only notes Washington is a general
and president.)

© 2018 The College Board.


Visit the College Board on the Web: www.collegeboard.org.

Downloaded by Loadster (kadenpizza66@gmail.com)


lOMoARcPSD|34158146

AP® UNITED STATES HISTORY


2018 SCORING GUIDELINES

Question 1 — Document-Based Question (continued)

Example of an acceptable explanation of the relevance of the document’s purpose:


• “In Document 3, after the impressment of Americans, Jay, a foreign diplomat sent to sort this issue out.
However, Jay’s treaty was a mockery of foreign affairs as they promised to amend part impressments
but refused to mention anything further. Jefferson rightly criticized the unpopular and ineffective
treaty.” (This example contributes toward a point for document sourcing because it connects the purpose
of Jefferson’s letter in understanding the failures of Jay’s Treaty.)

Example of an unacceptable explanation of the relevance of the document’s purpose:


• “This documents (5) purpose was to most likely figure out the American Wealthy upon early years
of trade.” (This example does not contribute toward a point for document sourcing because it does not
connect the purpose of the graph to an argument about change in United States foreign policy.)

Example of an acceptable explanation of the relevance of the historical situation of a document:


• “As the United States’ first president, George Washington believed that involvement with foreign
powers would detract from much-needed focus on the nation’s success, as seen in his neutrality
proclamation, a response to conflict between France and other European powers. Washington
sincerely hoped that United States foreign involvement would be limited. In his farewell address,
he explicitly warned of the danger that would come with making formal alliances with foreign powers.”
(This example contributes toward a point for document sourcing because it connects the situation
regarding neutrality to the conflicts in Europe.)

Example of an unacceptable explanation of the relevance of the historical situation of a document:


• “Some Americans like George Washington in Document 1 were neutral in the conflict between France
and Great Britain.” (This example does not contribute toward a point for document sourcing because it
does not connect the sentiments of Americans to change in United States foreign policy.)

Example of acceptable explanation of the relevance of the audience:


• “The fact that Thomas Jefferson (Document 3) was writing to political ally James Monroe, the US
minister to France at the time, highlights the disagreements about whether to side with Britain or
France. Jefferson, as a Democratic-Republican and Francophile, supported France.” (This example
contributes toward a point for document sourcing because it connects the audience for Jefferson’s letter
to internal conflict over the course of United States foreign policy.)

Example of unacceptable explanation of the relevance of the audience:


• “Document 5 has an audience of scholars formulating ideas in the 1970s when it was published.”
(This example does not contribute toward a point for document sourcing because it does not connect
the graph to the prompt nor an argument about change in United States foreign policy.)

© 2018 The College Board.


Visit the College Board on the Web: www.collegeboard.org.

Downloaded by Loadster (kadenpizza66@gmail.com)


lOMoARcPSD|34158146

AP® UNITED STATES HISTORY


2018 SCORING GUIDELINES

Question 1 — Document-Based Question (continued)

Demonstrating Complex Understanding


The response demonstrates a complex understanding, using evidence to corroborate, qualify, or modify
an argument that addresses the question.
• Explaining a nuance by exploring different ways (e.g., via conflicts abroad, via annexation of Native
American territory) in which United States foreign policy changed
• Explaining similarities and differences in the United States role in different parts of the world
• Explaining connections to other time periods such as United States expansionism in the late
nineteenth century
• Confirming the validity of the response’s argument by explaining how different documents corroborate
the argument in spite of differing perspectives among the authors
• Qualifying or modifying the argument by considering continuities in United States foreign policy
at this time

Example of demonstrating complex understanding:


• The response below includes several examples of historical complexity peppered throughout the essay that
use evidence to corroborate, qualify, or modify an argument about foreign policy. Here are some examples
from one response in multiple locations. “America was still establishing itself as a country because it
was new so it imported a lot from Britain, which is why America wanted to stay neutral in the
Revolution with France because Northern Federalists were relying on trade with Britain.” (explains a
nuance of an issue by analyzing multiple variables) “Adams request for isolationism is in line with the
Monroe Doctrine which say America does not want Europe interfering in the Western Hemisphere.”
(Explaining relevant and insightful connections with and across periods) “Britain still had a lot of forts
on the frontier which was supposed to be removed through Jay’s treaty. However, even after the treaty,
Britain didn’t accept America’s sovereignty at all and furthermore, France was angry over a British
treaty with America, which led to the XYZ affair with France.” (Confirming the validity of an argument
by corroborating multiple perspectives across themes) “Native Americans were very angry about the land
grab so they started getting guns from the British so they can rebel against Americans, particularly
Tecumseh, which escalated America’s tension with Great Britain.” (Continuity and change over time)

Example of unacceptably demonstrating complex understanding:


• The response below attempts to explain relevant and insightful connections within and across periods.
However, it is not demonstrating a complex understanding of the historical development that is the focus
of this essay, which is foreign policy from 1783–1828. This is merely a list of events in the 20th century
and beyond that are not connected to a sophisticated or nuanced argument. Merely making references to
other time periods in a synthetic fashion is not sufficient to earn the point. “Following the Declaration of
Independence and the United States separating itself from under Great Britain’s rule, there were many
conflicts to come. As American faced conflicts, such as WWI, WWII, Cold War, Great Depression,
and more, their foreign policy of the nation would inevitably adapt in order to be of assistance to the
betterment of the United States, For instance, with the current issues in the Middle Eastern nations
of Iraq and Iran, the United States are involved.”

© 2018 The College Board.


Visit the College Board on the Web: www.collegeboard.org.

Downloaded by Loadster (kadenpizza66@gmail.com)


lOMoARcPSD|34158146

AP® UNITED STATES HISTORY


2018 SCORING GUIDELINES

Question 2 — Long-Essay Question

Evaluate the extent to which trans-Atlantic exchanges affected colonial culture in British North America
in the period from 1700 to 1770.

Maximum Possible Points: 6

Points Rubric Notes


Thesis/Claim: Responds to the prompt The thesis must make a historically defensible claim
with a historically defensible that establishes a line of reasoning about how trans-
thesis/claim that establishes a line of Atlantic exchanges affected colonial culture in British
reasoning (1 point) North America in the period from 1700 to 1770.

To earn this point, the thesis must make a Examples that earn this point include:
claim that responds to the prompt rather • “From 1700-1770, the trans-Atlantic trade had
than restating or rephrasing the prompt. massive effects on American culture. A system of
The thesis must consist of one or more racial injustices through slavery was brought about
sentences located in one place, either in and the climate for the Revolution was made
the introduction or the conclusion. possible.” (The response suggests a line of argument
development.)
• “This interconnection of the two Hemispheres
A: Thesis/Claim

greatly impacted colonial culture between 1700 and


(0–1)

1770, as it changed demographics of the South, led


to sectional divisions, and allowed for the cultural
influence of Britain.” (The response establishes the
analytic categories for the argument.)
• “Although regional identities and cultures were
apparent before the trans-Atlantic trade such as the
life expectance and family values, the slave trade
helped further differentiate the regional differences
with changing daily lives and political/economic
differences.” (The response suggests a line of
argument development.)

© 2018 The College Board.


Visit the College Board on the Web: www.collegeboard.org.

Downloaded by Loadster (kadenpizza66@gmail.com)


lOMoARcPSD|34158146

AP® UNITED STATES HISTORY


2018 SCORING GUIDELINES

Question 2 — Long-Essay Question (continued)

Points Rubric Notes


Contextualization: Describes a broader To earn this point, the response must accurately
historical context relevant to the prompt. describe a context relevant to the ways in which
(1 point) trans-Atlantic exchanges affected colonial culture
in British North America in the period from 1700
To earn this point, the response must to 1770.
B: Contextualization

relate the topic of the prompt to broader


historical events, developments, or Examples of context might include the following, with
processes that occur before, during, or appropriate elaboration.
(0–1)

continue after the time frame of the • Mercantilism and economic ties to Great Britain
question. This point is not awarded for • The trans-Atlantic slave trade and the
merely a phrase or a reference. establishment of a social hierarchy (planter
aristocracy)
• Spread of Enlightenment and republican ideas
• The First Great Awakening and its effect upon
independent thought
• British taxation policies after the French and Indian
War influencing revolutionary ideals
Evidence: Provides specific examples of Examples of evidence used might include:
evidence relevant to the topic of the • Great Awakening (George Whitefield, Jonathan
prompt. (1 point) Edwards)
To earn the first point, the response must • Navigation Acts extended (Hat Act, Molasses Act,
identify specific historical examples of Iron Act)
evidence relevant to the topic of the • Salutary neglect
prompt. • Seven Years’ War (French and Indian War)
• Proclamation of 1763
OR • Pontiac’s Rebellion
C: Evidence (0–2)

• Stamp Act and repeal (Declaratory Act)


Supports an Argument: Supports an
• Sugar Act
argument in response to the prompt
• Townshend Acts
using specific and relevant examples of
• Triangular trade
evidence (2 points)
• Cash crops
To earn the second point, the response
• Slave trade, Middle Passage (slave rebellions –
must use specific historical evidence to
Stono Rebellion), Black Codes
support an argument in response to the
• Enlightenment effects on colonial thought
prompt.
• Boston Massacre

OR

Responses earn two points by using examples of


specific historical evidence to support an argument in
response to the prompt.

© 2018 The College Board.


Visit the College Board on the Web: www.collegeboard.org.

Downloaded by Loadster (kadenpizza66@gmail.com)


lOMoARcPSD|34158146

AP® UNITED STATES HISTORY


2018 SCORING GUIDELINES

Question 2 — Long-Essay Question (continued)

Points Rubric Notes


Historical Reasoning: Uses historical Examples of using historical reasoning to frame or
reasoning (e.g., comparison, causation, structure an argument might include:
continuity and change over time) to • Explaining how the Great Awakening affected
frame or structure an argument that colonial culture
addresses the prompt. (1 point) • Explaining how the Enlightenment affected
To earn the first point, the response must colonial culture
demonstrate the use of historical • Explaining how commercial exchanges of goods
reasoning to frame or structure an from Britain to the American colonies affected
argument, although the reasoning might colonial culture
be uneven or imbalanced. • Explaining how slavery and the slave trade affected
colonial culture
OR
OR
Complexity: Demonstrates a complex
understanding of the historical Demonstrating a complex understanding might include:
development that is the focus of the • Explaining a nuance of an issue by analyzing
D: Analysis and Reasoning (0–2)

prompt, using evidence to corroborate, multiple variables by, for example, examining
qualify, or modify an argument that several colonial regions (changing cultural
addresses the question. (2 points) identities between the regions)
• Explaining multiple causes or explaining both
To earn the second point, the response causes and effects, for example, explaining how
must demonstrate a complex colonial culture in the British North American
understanding. This can be accomplished colonies exhibited various features of independence
in a variety of ways, such as: and self-governance, while also expressing aspects
• Explaining a nuance of an issue by of loyalty to the British Crown
analyzing multiple variables • Explaining relevant and insightful connections
• Explaining both similarity and within and across periods, for example, examining
difference, or explaining both continuity the long-term effects of slavery on racial tensions
and change, or explaining multiple • Confirming the validity of an argument by
causes, or explaining both causes and corroborating multiple perspectives across themes
effects • Qualifying or modifying an argument by
• Explaining relevant and insightful considering diverse or alternative views or evidence
connections within and across periods by, for example, explaining how other factors were
• Confirming the validity of an argument more important than the trans-Atlantic exchanges
by corroborating multiple perspectives in shaping colonial culture
across themes • Explaining the various effects of slavery and the
• Qualifying or modifying an argument by slave trade on the development of regional identities
considering diverse or alternative views
or evidence
This understanding must be part of the
argument, not merely a phrase or
reference.

If response is completely blank, enter - - for all four score categories: A, B, C, and D

© 2018 The College Board.


Visit the College Board on the Web: www.collegeboard.org.

Downloaded by Loadster (kadenpizza66@gmail.com)


lOMoARcPSD|34158146

AP® UNITED STATES HISTORY


2018 SCORING GUIDELINES

Question 2 — Long-Essay Question (continued)

Scoring Notes

Introductory notes:
• Except where otherwise noted, each point of these rubrics is earned independently, e.g., a student
could earn a point for evidence without earning a point for thesis/claim.
• Accuracy: The components of these rubrics require that students demonstrate historically defensible
content knowledge. Given the timed nature of the exam, essays may contain errors that do not detract
from their overall quality, as long as the historical content used to advance the argument is accurate.
• Clarity: Exam essays should be considered first drafts and thus may contain grammatical errors.
Those errors will not be counted against a student unless they obscure the successful demonstration
of the content knowledge, skills, and practices described below.

Note: Student samples are quoted verbatim and may contain grammatical errors.

A. Thesis/Claim (0–1 points)

Responses earn one point by responding to the prompt with a historically defensible thesis that establishes
a line of reasoning about the topic. To earn this point, the thesis must make a claim that responds to the
prompt rather than simply restating or rephrasing the prompt. The thesis must suggest at least one main line
of argument development or establish the analytic categories of the argument.

The thesis must consist of one or more sentences located in one place, either in the introduction or the
conclusion.

Examples of acceptable theses:


• “The trans-Atlantic exchange of goods bolstered not only the economic system of British North
America, but the Spirits of the citizens. The goods brought to the colonies and the opportunity of trade
allowed the colonies to begin to prosper and directly affected the eventual development of the United
States of America.” (The response suggests a line of argument development.)
• “The trans-atlantic exchange also called the Columbian exchange fathered the culture of the Colonies,
giving them the building blocks for a democratic yet oppressive cuture.” (The response suggests a line
of argument development. The incorrect use of Columbian exchange is a minor error and does not detract
from the thesis statement.)
• “The Trans-Atlantic exchange played a massive role in the development of British colonial culture
because of how it effected their growth, culture, and eventual independence.” (The response establishes
the analytic categories for the argument.)

Examples of unacceptable theses:


• “In the years of 1700 to 1770 the trans-Atlantic exchanges heavily affected colonial culture in British
North America.” (This example largely restates the prompt).
• “In the time period of 1700 to 1770, the trans-Atlantic exchanges greatly affected colonial culture
in British North America as the exchange of plants, animals, good, and ideas influenced the rising
colonies. The colonies were not only getting consumer goods for the exchanges but they were getting
a taste of foreign language, music, dancing, social structure. All of those things influenced the colonial
culture.”(This response does not suggest a line of reasoning).
• “Trans-Atlantic exchanges affected colonial culture in British North America in many ways.”
(This example largely restates the prompt).

© 2018 The College Board.


Visit the College Board on the Web: www.collegeboard.org.

Downloaded by Loadster (kadenpizza66@gmail.com)


lOMoARcPSD|34158146

AP® UNITED STATES HISTORY


2018 SCORING GUIDELINES

Question 2 — Long-Essay Question (continued)

B. Contextualization (0–1 points)

Responses earn one point by describing a broader historical context relevant to the topic of the prompt.
To earn this point, the response must accurately and explicitly connect the context of the prompt to broader
historical events, developments, or processes that occurred before, during, or continued after the time frame
of the question. This point is not awarded for merely a phrase or reference.

To earn the point, the response must accurately describe a context relevant to the ways in which trans-Atlantic
exchanges affected colonial culture in British North America in the period from 1700 to 1770.

Examples might include the following, with appropriate elaboration:


• Mercantilism and economic ties to Great Britain
• The trans-Atlantic slave trade and the establishment of a social hierarchy (planter aristocracy)
• Spread of Enlightenment and republican ideas
• The First Great Awakening and its effect upon independent thought
• British taxation policies after the French and Indian War influencing revolutionary ideals

Example of acceptable contextualization:


• “Culturally America became more reliant on slaves and the plantation system was introduced.
The beginnings of racism towards blacks started with slavery. Culturally Americans did not see slaves
as humans. The slavery being introduced now would eventually create huge conflict that would result
in a Civil War.” (This example describes one broader historical context relevant to the topic of the prompt.)

Example of unacceptable contextualization:


• “The economies of most British North American colonies depended on farming in the 18th century.”
(This example in itself would not earn credit for contextualization because, even though it may be
historically accurate, it is not self-evidently relevant to the topic. To earn credit for contextualization,
this example would need to describe how the context offered is relevant to the topic.)

C. Evidence (0–2 points)

Evidence
Responses earn one point by providing at least two specific examples of evidence relevant to the topic of the
prompt. Responses can earn this point without earning the point for a thesis statement.

These examples of evidence must be different from the information used to earn the point for contextualization.
Typically, statements credited as contextualization will be more general statements that place an argument
or a significant portion of it in a broader context. Statements credited as evidence will typically be more
specific information.

© 2018 The College Board.


Visit the College Board on the Web: www.collegeboard.org.

Downloaded by Loadster (kadenpizza66@gmail.com)


lOMoARcPSD|34158146

AP® UNITED STATES HISTORY


2018 SCORING GUIDELINES

Question 2 — Long-Essay Question (continued)

Examples of evidence used might include:


• Great Awakening (George Whitefield, Jonathan Edwards)
• Navigation Acts extended (Hat Act, Molasses Act, Iron Act)
• Salutary Neglect
• French and Indian War
• Proclamation of 1763
• Pontiac’s Rebellion
• Stamp Act and repeal (Declaratory Act)
• Sugar Act
• Townshend Acts
• Triangular trade
• Cash crops
• Slave trade, Middle Passage (slave rebellions – Stono Rebellion), Black Codes
• Enlightenment effects on colonial thought
• Boston Massacre

Example of acceptably providing evidence relevant to the topic of the prompt


• “Protestant minister George Whitefield was an important leader who participated in the Great
Awakening in both England and America.” (This example cites evidence relevant to the topic, so it would
count for the first evidence point, but it does not use that evidence to support an argument about how trans-
Atlantic exchanges affected United States culture, so it would not earn the second point.)

Example of unacceptably providing evidence relevant to the topic of the prompt


• “Alexander Hamilton convinced George Washington to support France in French wars against Great
Britain.” (This response would not earn credit for evidence because it is outside of the time period and
historically inaccurate.)

OR

Supports an Argument
Responses earn two points if they support an argument in response to the prompt using specific and relevant
examples of evidence.

Example of acceptable use of evidence to support an argument


• “By preaching first in England and then traveling across the Atlantic to America and preaching there,
Protestant minister George Whitefield helped transfer religious ideas from Europe to the American
colonies and changed American culture.” (This example gives specific historical evidence that supports
an argument in response to the prompt.)

Example of unacceptable use of evidence to support an argument


• “Protestant minister George Whitefield was an important leader who participated in the Great
Awakening.” (This example cites evidence relevant to the topic, so it would count for the first evidence
point, but it does not use that evidence to support an argument about how trans-Atlantic exchanges
affected United States culture, so it would not earn the second point.)

© 2018 The College Board.


Visit the College Board on the Web: www.collegeboard.org.

Downloaded by Loadster (kadenpizza66@gmail.com)


lOMoARcPSD|34158146

AP® UNITED STATES HISTORY


2018 SCORING GUIDELINES

Question 2 — Long-Essay Question (continued)

D. Analysis and Reasoning (0–2 points)

Historical Reasoning
Responses earn one point by using historical reasoning to frame or structure an argument that addresses the
prompt. To earn this point, the response must demonstrate the use of historical reasoning to frame or structure
an argument, although the reasoning might be uneven or imbalanced.

Examples of using historical reasoning might include:


• Explaining how the Great Awakening affected colonial culture
• Explaining how the Enlightenment affected colonial culture
• Explaining how commercial exchanges of goods from Britain to the American colonies affected
colonial culture
• Explaining how slavery and the slave trade affected colonial culture

Example of acceptable use of historical reasoning:


• “European Enlightenment ideas such as self-rule and the social contract migrated across the Atlantic
from Europe to America in the 18th century and helped create the intellectual and cultural basis for the
American Revolution.” (This example earns the point for historical reasoning because it highlights
movement and a historical change. The use of terms like “migrated” and “helped create” demonstrates
change over time.)

Example of unacceptable use of historical reasoning:


• “The Enlightenment was a very important factor in 18th century America.” (This sentence would not
earn the point for historical reasoning because the language does not indicate change, continuity,
comparison, or causation.)
OR

Complexity
Responses earn two points for demonstrating a complex understanding of the topic, using evidence to
corroborate, qualify, or modify that argument.

Demonstrating complex understanding might include:


• Explaining a nuance of an issue by analyzing multiple variables by, for example, examining several
colonial regions (changing cultural identities between the regions)
• Explaining multiple causes or explaining both causes and effects, for example, explaining how colonial
culture in the British North American colonies exhibited various features of independence and self-
governance, while also expressing aspects of loyalty to the British Crown
• Explaining relevant and insightful connections within and across periods, for example, examining the
long-term effects of slavery on racial tensions
• Confirming the validity of an argument by corroborating multiple perspectives across themes
• Qualifying or modifying an argument by considering diverse or alternative views or evidence by, for
example, explaining how other factors were more important than the trans-Atlantic exchanges in
shaping colonial culture
• Explaining the various effects of slavery and the slave trade on the development of regional identities

This understanding must be part of the argument, not merely a phrase or reference.

© 2018 The College Board.


Visit the College Board on the Web: www.collegeboard.org.

Downloaded by Loadster (kadenpizza66@gmail.com)


lOMoARcPSD|34158146

AP® UNITED STATES HISTORY


2018 SCORING GUIDELINES

Question 2 — Long-Essay Question (continued)

Example of acceptable demonstration of a complex understanding


• “This brought political power to the South and allowed for the creation of a planters’ aristocracy,
This class system, developed primarily in the mid-1700s would go on to rule Dixieland through
the 19th century and being a cause of the Civil War.” (This example, through the growth of slavery
in the South, demonstrates a complex understanding of the historical development that is the focus
of the prompt by making insightful connections across periods.)

Example of unacceptable demonstration of a complex understanding


• “By 1770 most colonists were angry with Great Britain and either wanted them to back or wanted
independence from them. This feeling was much like the south’s feelings before the civil war where
they felt the north was infringing on their livelihood. They also felt they may thrive if they separated,
so they succeeded from the union much like America did from Britain.” (This example attempts to make
a connection between the two time periods but provides an inaccurate connection.)

© 2018 The College Board.


Visit the College Board on the Web: www.collegeboard.org.

Downloaded by Loadster (kadenpizza66@gmail.com)


lOMoARcPSD|34158146

AP® UNITED STATES HISTORY


2018 SCORING GUIDELINES

Question 3 — Long-Essay Question

Evaluate the extent to which immigration affected United States culture in the period from 1840 to 1898.

Maximum Possible Points: 6

Points Rubric Notes


Thesis/Claim: Responds to the prompt The thesis must make a historically defensible claim
with a historically defensible that establishes a line of reasoning about how
thesis/claim that establishes a line of immigration affected United States culture in the
reasoning (1 point) period from 1840 to 1898.
A: Thesis/Claim

To earn this point, the thesis must make Examples that earn this point include:
(0–1)

a claim that responds to the prompt rather • “Immigration greatly affected culture in the United
than restating or rephrasing the prompt. States from 1840 to 1898 by causing manufacturing
The thesis must consist of one or more to increase, discontent of Americans, and an overall
sentences located in one place, either in change in the population, especially in cities.”
the introduction or the conclusion. • “Immigration in the period 1840 to 1898 led to
increased prevalence in United States culture of
nativism, discrimination, and racism.”
Contextualization: Describes a broader To earn this point, the response must accurately
historical context relevant to the prompt. describe a context relevant to the ways in which
B: Contextualization

(1 point) immigration affected United States culture in the


period from 1840 to 1898.
To earn this point, the response must
(0–1)

relate the topic of the prompt to broader Examples of context might include the following,
historical events, developments, with appropriate elaboration.
or processes that occur before, during, • The industrialization of the United States
or continue after the time frame of the • Economic opportunity in the United States
question. This point is not awarded for • Opportunities provided by Manifest Destiny
merely a phrase or a reference.

© 2018 The College Board.


Visit the College Board on the Web: www.collegeboard.org.

Downloaded by Loadster (kadenpizza66@gmail.com)


lOMoARcPSD|34158146

AP® UNITED STATES HISTORY


2018 SCORING GUIDELINES

Question 3 — Long-Essay Question (continued)

Points Rubric Notes


Evidence: Provides specific examples of Examples of evidence used might include:
evidence relevant to the topic of the • Irish and German immigration, 1840s–1850s
prompt. (1 point) • Southern, eastern, and central European immigration
To earn the first point, the response must in the late nineteenth century
identify specific historical examples of • Chinese Exclusion Act of 1882
evidence relevant to the topic of the • Know-Nothing Party
C: Evidence (0–2)

prompt. • Urban political machines and bosses


• Chinatowns, Little Italy, Germantowns
OR • Nativism; “no Irish need apply”
• Angel and Ellis Island
Supports an Argument: Supports an
• Hull House, 1889
argument in response to the prompt
• Jacob Riis
using specific and relevant examples of
• Transcontinental railroad
evidence (2 points)
To earn the second point, the response
must use specific historical evidence to
support an argument in response to the
prompt.

© 2018 The College Board.


Visit the College Board on the Web: www.collegeboard.org.

Downloaded by Loadster (kadenpizza66@gmail.com)


lOMoARcPSD|34158146

AP® UNITED STATES HISTORY


2018 SCORING GUIDELINES

Question 3 — Long-Essay Question (continued)

Points Rubric Notes


Historical Reasoning: Uses historical Examples of using historical reasoning to frame or
reasoning (e.g., comparison, causation, structure an argument might include:
continuity and change over time) to • Explaining how immigration fostered the creation of
frame or structure an argument that ethnic communities and how ethnic cultures shaped
addresses the prompt. (1 point) other cultures
To earn the first point, the response must • Explaining how immigration contributed to the rise
demonstrate the use of historical of nativist ideas and nativism
reasoning to frame or structure an • Explaining how immigration contributed to
argument, although the reasoning might urbanization and urban challenges
be uneven or imbalanced. • Explaining how immigration contributed to a
working class culture
OR
OR
Complexity: Demonstrates a complex
understanding of the historical Demonstrating a complex understanding might include:
development that is the focus of the • Explaining a nuance of an issue by analyzing
D: Analysis and Reasoning (0–2)

prompt, using evidence to corroborate, multiple variables by, for example, examining
qualify, or modify an argument that multiple immigrant groups
addresses the question. (2 points) • Explaining multiple causes or explaining both causes
and effects by, for example, explaining
To earn the second point, the response how the influence of immigration generated nativist
must demonstrate a complex sentiment in the United States, while
understanding. This can be accomplished at the same time creating an increasingly ethnically
in a variety of ways, such as: diverse culture
• Explaining a nuance of an issue by • Explaining relevant and insightful connections
analyzing multiple variables within and across periods by making comparisons
• Explaining both similarity and with the effects of immigration after 1898
difference, or explaining both continuity • Confirming the validity of an argument by
and change, or explaining multiple corroborating multiple perspectives across various
causes, or explaining both causes and immigrant groups and/or various American political
effects and social platforms.
• Explaining relevant and insightful • Qualifying or modifying an argument by considering
connections within and across periods diverse or alternative views or evidence by, for
• Confirming the validity of an argument example, arguing and explaining how other factors
by corroborating multiple perspectives were more important than immigration in shaping
across themes United States culture.
• Qualifying or modifying an argument by
considering diverse or alternative views
or evidence
This understanding must be part of the
argument, not merely a phrase or
reference.

If response is completely blank, enter - - for all four score categories: A, B, C, and D

© 2018 The College Board.


Visit the College Board on the Web: www.collegeboard.org.

Downloaded by Loadster (kadenpizza66@gmail.com)


lOMoARcPSD|34158146

AP® UNITED STATES HISTORY


2018 SCORING GUIDELINES

Question 3 — Long-Essay Question (continued)

Scoring Notes

Introductory notes:
• Except where otherwise noted, each point of these rubrics is earned independently, e.g., a student
could earn a point for evidence without earning a point for thesis/claim.
• Accuracy: The components of these rubrics require that students demonstrate historically defensible
content knowledge. Given the timed nature of the exam, essays may contain errors that do not detract
from their overall quality, as long as the historical content used to advance the argument is accurate.
• Clarity: Exam essays should be considered first drafts and thus may contain grammatical errors.
Those errors will not be counted against a student unless they obscure the successful demonstration
of the content knowledge, skills, and practices described below.

Note: Student samples are quoted verbatim and may contain grammatical errors.

A. Thesis/Claim (0–1 points)

Responses earn one point by responding to the prompt with a historically defensible thesis that establishes
a line of reasoning about the topic. To earn this point, the thesis must make a claim that responds to the
prompt rather than simply restating or rephrasing the prompt. The thesis must suggest at least one main line
of argument development or establish the analytic categories of the argument.

The thesis must consist of one or more sentences located in one place, either in the introduction or
the conclusion.

Examples of acceptable theses:


• “Immigration greatly affected culture in the United States from 1840 to 1898 by causing manufacturing
to increase, discontent of Americans, and an overall change in the population, especially in cities.”
(The response suggests a line of argument development.)
• “This influx of immigration had a significant effect on U.S. culture from 1840-1898 as it caused
a reaction in nativist movements, created a new labor force for manufacturing, and ultimately led
to the formation of unions.” (The response establishes the analytic categories for the argument.)
• “Immigration in the period 1840 to 1898 led to increased prevalence in United States culture of
nativism, discrimination, and racism.” (The response establishes the analytic categories for the
argument.)

Example of unacceptable thesis:


• “The most significant way that immigration shaped American culture was the arrival of more people.”
(This response is too generic to earn credit as a thesis).
• “Immigration in the period 1840 to 1898 affected United States culture to a large extent.” (This example
largely restates the prompt).

© 2018 The College Board.


Visit the College Board on the Web: www.collegeboard.org.

Downloaded by Loadster (kadenpizza66@gmail.com)


lOMoARcPSD|34158146

AP® UNITED STATES HISTORY


2018 SCORING GUIDELINES

Question 3 — Long-Essay Question (continued)

B. Contextualization (0–1 points)

Responses earn one point by describing a broader historical context relevant to the topic of the prompt.
To earn this point, the response must accurately and explicitly connect the context of the prompt to broader
historical events, developments, or processes that occurred before, during, or continued after the time frame
of the question. This point is not awarded for merely a phrase or reference.

To earn the point, the response must accurately describe a context relevant to the ways in which immigration
affected United States culture in the period from 1840 to 1898.

Examples might include the following, with appropriate elaboration:


• The industrialization of the United States
• Economic opportunity in the United States

Example of acceptable contextualization:


• “The Gilded Age was a time of rapid industrialization that helped attract many immigrants.
Because native born Americans looked down on these jobs, Eastern and Southern European
immigrants, who arrived with little money, took these unskilled factory jobs and dangerous
manufacturing positions in construction and steel. Immigrants in this sector of the labor force
increased the wealth and power of the industrial giants, who were protected by the federal
government through the lack of regulation.” (This example describes one broader historical context
relevant to the topic of the prompt.)

Example of unacceptable contextualization:


• “Political corruption was widespread in the 1880s and 1890s.” (This example in itself would not earn
credit for contextualization because, even though it may be historically accurate, it is not self-evidently
relevant to the topic. To earn credit for contextualization, this example would need to describe how the
context offered is relevant to the topic.)

C. Evidence (0–2 points)

Evidence
Responses earn one point by providing at least two specific examples of evidence relevant to the topic
of the prompt. Responses can earn this point without earning the point for a thesis statement.

These examples of evidence must be different from the information used to earn the point for contextualization.
Typically, statements credited as contextualization will be more general statements that place an argument or
a significant portion of it in a broader context. Statements credited as evidence will typically be more specific
information.

© 2018 The College Board.


Visit the College Board on the Web: www.collegeboard.org.

Downloaded by Loadster (kadenpizza66@gmail.com)


lOMoARcPSD|34158146

AP® UNITED STATES HISTORY


2018 SCORING GUIDELINES

Question 3 — Long-Essay Question (continued)

Examples of evidence used might include:


• Irish and German immigration, 1840s–1850s
• Southern, eastern, and central European immigration in the late nineteenth century
• Chinese Exclusion Act of 1882
• Know-Nothing Party
• Urban political machines and bosses
• Chinatowns, Little Italy, Germantowns
• Nativism; “no Irish need apply”
• Angel and Ellis Island
• Hull House, 1889
• Jacob Riis
• Transcontinental Railroad

Example of acceptably providing evidence relevant to the topic of the prompt


• “Many Irish people immigrated to the United States in the 1840s and 1850s.” (This example cites
evidence relevant to the topic, so it would count towards the first evidence point, but it does not use that
evidence to support an argument about how immigration affected United States culture, so it would not
count towards the second point.)

Example of unacceptably providing evidence relevant to the topic of the prompt


• “Millions of people came to the United States looking for jobs.” (This response would not earn credit
for evidence because it is too vague.)

OR

Supports an Argument
Responses earn two points if they support an argument in response to the prompt using specific and relevant
examples of evidence.

Example of acceptable use of evidence to support an argument


• “Large-scale Irish immigration to the United States in the 1840s and 1850s introduced new cultural
traditions to the United States and expanded the influence of the Catholic religion. This also prompted
nativist antagonism toward Irish immigrants.” (This example connects a piece of evidence—Irish
immigration—to a number of cultural trends at the time.)

Example of unacceptable use of evidence to support an argument


• “Many Irish people immigrated to the United States in the 1840s and 1850s.” (This example cites
evidence relevant to the topic, so it would count towards the first evidence point, but it does not use that
evidence to support an argument about how immigration affected United States culture, so it would not
count towards
the second point.)

© 2018 The College Board.


Visit the College Board on the Web: www.collegeboard.org.

Downloaded by Loadster (kadenpizza66@gmail.com)


lOMoARcPSD|34158146

AP® UNITED STATES HISTORY


2018 SCORING GUIDELINES

Question 3 — Long-Essay Question (continued)

D. Analysis and Reasoning (0–2 points)

Historical Reasoning
Responses earn one point by using historical reasoning to frame or structure an argument that addresses
the prompt. To earn this point, the response must demonstrate the use of historical reasoning to frame or
structure an argument, although the reasoning might be uneven or imbalanced.

Examples of using historical reasoning might include:


• Explaining how immigration fostered the creation of ethnic communities and how ethnic cultures
shaped other cultures
• Explaining how immigration contributed to the rise of nativist ideas and nativism
• Explaining how immigration contributed to urbanization and urban challenges
• Explaining how immigration contributed to a working-class culture

Example of acceptable use of historical reasoning:


• “Immigration led to the development of ethnic communities in cities such as the Chinatowns that
emerged in New York City and San Francisco.” (This example earns the point for historical reasoning
because it highlights a historical change. The use of terms like “development” and “emerged” demonstrates
change over time.)

Example of unacceptable use of historical reasoning:


• “Immigration was very important in the late 19th century.” (This sentence would not earn the point for
historical reasoning because the language does not indicate change, continuity, comparison, or causation.)

Complexity
Responses earn two points for demonstrating a complex understanding of the topic, using evidence to
corroborate, qualify, or modify that argument.

Demonstrating complex understanding might include:


• Explaining a nuance of an issue by analyzing multiple variables by, for example, examining multiple
immigrant groups
• Explaining multiple causes or explaining both causes and effects by, for example, explaining how
the influence of immigration generated nativist sentiment in the United States, while at the same time
creating an increasingly ethnically diverse culture
• Explaining relevant and insightful connections within and across periods by making comparisons with
the effects of immigration after 1898
• Confirming the validity of an argument by corroborating multiple perspectives across themes
• Qualifying or modifying an argument by considering diverse or alternative views or evidence by,
for example, arguing and explaining how other factors were more important than immigration in
shaping United States culture.

This understanding must be part of the argument, not merely a phrase or reference.

© 2018 The College Board.


Visit the College Board on the Web: www.collegeboard.org.

Downloaded by Loadster (kadenpizza66@gmail.com)


lOMoARcPSD|34158146

AP® UNITED STATES HISTORY


2018 SCORING GUIDELINES

Question 3 — Long-Essay Question (continued)

Example of acceptable demonstration of a complex understanding


• With additional such examples throughout essay, this example demonstrates a complex understanding
through relevant and insightful connections across time periods.
“Because of the changes brought about by immigration, a profound divide between Americans
emerged. Many Americans sought to assist and assimilate immigrants through educational and
training programs, while others increased participation in discriminatory practices. This division led
to progressivism on one side and increased calls for immigration restriction on the other. Despite the
efforts of progressives, nativism prevailed through the passage of quotas in the early 20th century and
the rise of the Ku Klux Klan’s targeting of immigrants.”

Example of unacceptable demonstration of a complex understanding


• This example does not demonstrate a complex understanding of the evidence in support of the argument.
“Immigration was very important because it significantly changed America.”

© 2018 The College Board.


Visit the College Board on the Web: www.collegeboard.org.

Downloaded by Loadster (kadenpizza66@gmail.com)


lOMoARcPSD|34158146

AP® UNITED STATES HISTORY


2018 SCORING GUIDELINES

Question 4 — Long-Essay Question

Evaluate the extent to which sustained economic growth affected United States culture in the period from
1940 to 1970.

Maximum Possible Points: 6

Points Rubric Notes


Thesis/Claim: Responds to the prompt The thesis must make a historically defensible claim
with a historically defensible that establishes a line of reasoning about how
thesis/claim that establishes a line of sustained economic growth affected United States
reasoning (1 point) culture in the period from 1940 to 1970.

To earn this point, the thesis must make a Examples that earn this point include:
A: Thesis/Claim

claim that responds to the prompt rather • “America’s economic success would directly
than restating or rephrasing the prompt. correlate to changes in lifestyle, social norms,
(0–1)

The thesis must consist of one or more standard of living, and would contribute to literacy
sentences located in one place, either in and sociopolitical movements.”
the introduction or the conclusion. • “The sustained economic growth affected the United
States culture by forcing the extreme conformity in
the 1950s, the exploration of new concepts, and the
uprising of civil rights.”
(These responses make historically defensible claims
that address the prompt.)

Contextualization: Describes a broader To earn this point, the response must accurately
historical context relevant to the prompt. describe a context relevant to the ways in which
(1 point) sustained economic growth affected United States
B: Contextualization

culture in the period from 1940 to 1970.


To earn this point, the response must
(0–1)

relate the topic of the prompt to broader Examples of context might include the following,
historical events, developments, or with appropriate elaboration.
processes that occur before, during, or • Postwar liberalism
continue after the time frame of the • The Cold War
question. This point is not awarded for • The Great Depression
merely a phrase or a reference. • Space Race
• World War II

© 2018 The College Board.


Visit the College Board on the Web: www.collegeboard.org.

Downloaded by Loadster (kadenpizza66@gmail.com)


lOMoARcPSD|34158146

AP® UNITED STATES HISTORY


2018 SCORING GUIDELINES

Question 4 — Long-Essay Question (continued)

Points Rubric Notes


Evidence: Provides specific examples of Examples of evidence used might include:
evidence relevant to the topic of the • Baby boom
prompt. (1 point) • Great Society
To earn the first point, the response must • Levittown
identify specific historical examples of • Sun Belt
evidence relevant to the topic of the • Interstate Highway System
C: Evidence (0–2)

prompt. • Suburbanization
• Counterculture – Beatniks - Hippies
OR • Rock n roll
• Civil Rights movement
Supports an Argument: Supports an
• Emergence of distinct teenage youth culture
argument in response to the prompt
• American Dream
using specific and relevant examples of
• Proxy wars
evidence (2 points)
• Women’s rights
To earn the second point, the response
• Appliances, cars
must use specific historical evidence to
• White picket fences
support an argument in response to the
• Advertising in the context of a consumer culture
prompt.
• “Keeping up with the Joneses”

© 2018 The College Board.


Visit the College Board on the Web: www.collegeboard.org.

Downloaded by Loadster (kadenpizza66@gmail.com)


lOMoARcPSD|34158146

AP® UNITED STATES HISTORY


2018 SCORING GUIDELINES

Question 4 — Long-Essay Question (continued)

Points Rubric Notes


Historical Reasoning: Uses historical Examples of using historical reasoning to frame or
reasoning (e.g., comparison, causation, structure an argument might include:
continuity and change over time) to • Explaining how economic growth contributed to
frame or structure an argument that suburban culture
addresses the prompt. (1 point) • Explaining how sustained economic growth
To earn the first point, the response must contributed to popular demands for expanded social
demonstrate the use of historical or political rights
reasoning to frame or structure an
argument, although the reasoning might OR
be uneven or imbalanced.
Demonstrating a complex understanding might include:
OR • Explaining a nuance of an issue by analyzing
multiple variables by, for example, exploring
Complexity: Demonstrates a complex racial or ethnic variations in United States culture
understanding of the historical or by exploring how economic growth affected
development that is the focus of the groups differently
D: Analysis and Reasoning (0-2)

prompt, using evidence to corroborate, • Explaining multiple causes such as several particular
qualify, or modify an argument that aspects of economic growth or explaining both
addresses the question. (2 points) causes and effects
• Explaining relevant and insightful connections
To earn the second point, the response within and across periods such as making
must demonstrate a complex comparisons with economic growth in the 1920s
understanding. This can be accomplished • Confirming the validity of an argument by
in a variety of ways, such as: corroborating multiple perspectives across themes
• Explaining a nuance of an issue by by demonstrating that the development of the
analyzing multiple variables counterculture was spurred by a general culture
• Explaining both similarity and of conformity, itself a product of sustained
difference, or explaining both continuity economic growth
and change, or explaining multiple • Qualifying or modifying an argument by considering
causes, or explaining both causes diverse or alternative views or evidence by,
and effects for example, arguing how other factors were more
• Explaining relevant and insightful important than sustained economic growth
connections within and across periods in shaping United States culture in the period
• Confirming the validity of an argument 1940 to 1970
by corroborating multiple perspectives
across themes
• Qualifying or modifying an argument by
considering diverse or alternative views
or evidence
This understanding must be part of the
argument, not merely a phrase or
reference.

If response is completely blank, enter - - for all four score categories: A, B, C, and D

© 2018 The College Board.


Visit the College Board on the Web: www.collegeboard.org.

Downloaded by Loadster (kadenpizza66@gmail.com)


lOMoARcPSD|34158146

AP® UNITED STATES HISTORY


2018 SCORING GUIDELINES

Question 4 — Long-Essay Question (continued)

Scoring Notes

Introductory notes:
• Except where otherwise noted, each point of these rubrics is earned independently, e.g., a student
could earn a point for evidence without earning a point for thesis/claim.
• Accuracy: The components of these rubrics require that students demonstrate historically defensible
content knowledge. Given the timed nature of the exam, essays may contain errors that do not detract
from their overall quality, as long as the historical content used to advance the argument is accurate.
• Clarity: Exam essays should be considered first drafts and thus may contain grammatical errors.
Those errors will not be counted against a student unless they obscure the successful demonstration
of the content knowledge, skills, and practices described below.

Note: Student samples are quoted verbatim and may contain grammatical errors.

A. Thesis/Claim (0–1 points)

Responses earn one point by responding to the prompt with a historically defensible thesis that establishes
a line of reasoning about the topic. To earn this point, the thesis must make a claim that responds to the
prompt rather than simply restating or rephrasing the prompt. The thesis must suggest at least one main line
of argument development or establish the analytic categories of the argument.

The thesis must consist of one or more sentences located in one place, either in the introduction or the
conclusion.

Examples of acceptable theses:


• “In the period between 1940 to 1970 enormous sustained economic growth had fundamentally changed
the culture of the United States in regards to living standards and expectations, who should be able
to have access to our democratic society, and what the American lifestyle meant.” (This response is
acceptable because it presents a historically defensible thesis that identifies three ways in which sustained
economic growth changed American culture.)
• “From 1940 to 1970 the country’s affluence and overall growth forever changed cultural issues
regarding civil rights, the role of women, and the lifestyle of Americans.” (This response is acceptable
because it presents a historically defensible thesis that establishes a line of reasoning about the topic.)

Example of unacceptable theses:


• “During the time between 1940-1970, America learned to thrive and bring citizens with high benefits
to improve society economically and socially.” (This response is unacceptable because it does not make
any claim or statement about the impact of sustained economic growth on American culture).
• “Sustained economic growth greatly affected United States culture from 1940 to 1970.” (This response
essentially re-states the question.)

© 2018 The College Board.


Visit the College Board on the Web: www.collegeboard.org.

Downloaded by Loadster (kadenpizza66@gmail.com)


lOMoARcPSD|34158146

AP® UNITED STATES HISTORY


2018 SCORING GUIDELINES

Question 4 — Long-Essay Question (continued)

B. Contextualization (0–1 points)

Responses earn one point by describing a broader historical context relevant to the topic of the prompt.
To earn this point, the response must accurately and explicitly connect the context of the prompt to broader
historical events, developments, or processes that occurred before, during, or continued after the time frame
of the question. This point is not awarded for merely a phrase or reference.

To earn the point, the response must accurately describe a context relevant to the ways in which sustained
economic growth affected United States culture in the period from 1940 to 1970.

Examples might include the following, with appropriate elaboration:


• Postwar liberalism
• The Cold War
• The Great Depression
• Space Race
• World War II

Example of acceptable contextualization:


• “This occurred within the context of the Cold War as tension kept mounting domestically.
Consequently, the American people would turn towards their consumerism and home lives in order
to escape the growing fear of the threat that the Soviet Union presented.” (This example describes one
broader historical context relevant to the topic of the prompt.)

Example of unacceptable contextualization:


• “Because of WWII, the Great Depression had ended and America emerged economically stable.”
(This example in itself would not earn credit for contextualization because, while it mentions the Great
Depression as context for the economic growth, it is merely a phrase or a reference.)

C. Evidence (0–2 points)

Responses earn one point by providing at least two specific examples of evidence relevant to the topic of the
prompt. Responses can earn this point without earning the point for a thesis statement.

These examples of evidence must be different from the information used to earn the point for contextualization.
Typically, statements credited as contextualization will be more general statements that place an argument
or a significant portion of it in a broader context. Statements credited as evidence will typically be more
specific information.

© 2018 The College Board.


Visit the College Board on the Web: www.collegeboard.org.

Downloaded by Loadster (kadenpizza66@gmail.com)


lOMoARcPSD|34158146

AP® UNITED STATES HISTORY


2018 SCORING GUIDELINES

Question 4 — Long-Essay Question (continued)

Examples of evidence used might include:


• Baby boom
• Great Society
• Levittown
• Sun Belt
• Interstate Highway System
• Suburbanization
• Counterculture – Beatniks - Hippies
• Rock n roll
• Civil Rights movement
• Emergence of distinct teenage youth culture
• American Dream
• Proxy wars
• Women’s rights
• Appliances, cars
• White picket fences
• Advertising in the context of a consumer culture
• “Keeping up with the Joneses”
• Women’s Rights
• Appliances, Cars
• White Picket Fence
• Advertising
• “Keeping up with the Joneses

Example of acceptably providing evidence relevant to the topic of the prompt


• “In the 1950s for the first time a bloated middle class could afford housing in large, tracted suburbs,
buy appliances with wages that paid well, and afford to send themselves and their kids to college.”
(This response cites evidence – suburbs, appliances, college - relevant to the topic. Because the response
to not go on to use the evidence to support its argument, it did not earn the second evidence point.)

Example of unacceptably providing evidence relevant to the topic of the prompt


• “With the help of Woodrow Wilson, the United States brought the unemployment rate back to 2
percent.” (This response would not earn credit for evidence because it is outside of the time period and
historically inaccurate.)

OR

Supports an Argument
Responses earn two points if they support an argument in response to the prompt using specific and relevant
examples of evidence.

Example of acceptable use of evidence to support an argument


• “Developments in media made possible by a growing economy led to the definition of American culture
and the popularization of American culture between 1940 and 1970.” (As part of response that cites
the role of television – it says, for example “Soon shows reinforcing the nuclear family and the roles of each
person, such as ‘Father Knows Best,’ were on every TV screen - and movies in creating a culture of
conformity, this example uses the evidence to support its argument.)

© 2018 The College Board.


Visit the College Board on the Web: www.collegeboard.org.

Downloaded by Loadster (kadenpizza66@gmail.com)


lOMoARcPSD|34158146

AP® UNITED STATES HISTORY


2018 SCORING GUIDELINES

Question 4 — Long-Essay Question (continued)

Example of unacceptable use of evidence to support an argument


• “America being financially prosperous at the time created the Berlin Airlift to keep Germany on the
democratic side of Cold War aggression.” (This response cites evidence relevant to the topic, so it would
count for the first evidence point, but it does not use that evidence to support an argument about changes
in United States culture, so it would not earn the second point.)

D. Analysis and Reasoning (0–2 points)

Historical Reasoning
Responses earn one point by using historical reasoning to frame or structure an argument that addresses
the prompt. To earn this point, the response must demonstrate the use of historical reasoning to frame
or structure an argument, although the reasoning might be uneven or imbalanced.

Examples of using historical reasoning might include:


• Explaining how economic growth contributed to suburban culture
• Explaining how sustained economic growth contributed to popular demands for expanded social
or political rights

Example of acceptable use of historical reasoning:


• “Instead of usually living in cities or rurally, a migration to the suburbs started. Modern cars, education
and technology made this possible.” (This response earns the point for historical reasoning because it
explains causation for a cultural change – growth of suburbs – related to sustained economic growth
(1940-1970).)

Example of unacceptable use of historical reasoning:


• “The reason that all of these different cultural changes was as a result from economic prosperity.”
(This response would not earn the point for historical reasoning because, while it attempts to identify
causation, it does not actually make the connection.)

OR

Complexity
Responses earn two points for demonstrating a complex understanding of the topic, using evidence
to corroborate, qualify, or modify that argument.

© 2018 The College Board.


Visit the College Board on the Web: www.collegeboard.org.

Downloaded by Loadster (kadenpizza66@gmail.com)


lOMoARcPSD|34158146

AP® UNITED STATES HISTORY


2018 SCORING GUIDELINES

Question 4 — Long-Essay Question (continued)

Demonstrating complex understanding might include:


• Explaining a nuance of an issue by analyzing multiple variables by, for example, exploring racial
or ethnic variations in United States culture or by exploring how economic growth affected groups
differently
• Explaining multiple causes such as several particular aspects of economic growth or explaining both
causes and effects
• Explaining relevant and insightful connections within and across periods such as making comparisons
with economic growth in the 1920s
• Confirming the validity of an argument by corroborating multiple perspectives across themes by
demonstrating that the development of the counterculture was spurred by a general culture of
conformity, itself a product of sustained economic growth
• Qualifying or modifying an argument by considering diverse or alternative views or evidence by,
for example, arguing how other factors were more important than sustained economic growth in
shaping United States culture in the period 1940 to 1970

This understanding must be part of the argument, not merely a phrase or reference.

Example of acceptable demonstration of a complex understanding


(Part of the argument presented in this response involved how American culture, backed by sustained
economic growth, had spread across the world. This example earned the point for complexity because
it modifies that argument by recognizing that American culture, even as it was widely embraced abroad,
also adopted aspects of other cultures.)
• “As our exposure to cultural exchange increased, the American culture began to adopt other foreign
cultures, such as Japanese media and European clothing brands and food and music, which only
served to further enrich our culture.”

Example of unacceptable demonstration of a complex understanding


(This response is unacceptable because, while it attempts to explain nuance on the issue of conformity,
it attributes the cause incorrectly.)
• “The jobs were similar, and so were the hours, making most American families fall into with
each other.”

© 2018 The College Board.


Visit the College Board on the Web: www.collegeboard.org.

Downloaded by Loadster (kadenpizza66@gmail.com)


lOMoARcPSD|34158146

Scoring Worksheet

The following provides a scoring worksheet and conversion table


used for calculating a composite score of the exam.

Downloaded by Loadster (kadenpizza66@gmail.com)


lOMoARcPSD|34158146

2018 AP United States History Scoring Worksheet

Section I, Part A: Multiple Choice

× 1.0181 =
Number Correct Weighted Section I, Part A
(out of 55) Score (Do not round)

Section I, Part B: Short Answer

Question 1 × 3.1111 =
(out of 3) (Do not round)

Question 2 × 3.1111 =
(out of 3) (Do not round)

Question 3 × 3.1111 =
(out of 3) (Do not round)

Section II: Free Response

DBQ × 5.0000 =
(out of 7) (Do not round)

Choice Essay × 3.5000 =


(out of 6) (Do not round)

Sum =
Weighted
Section I, Part B
and Part II Score
(Do not round)

Composite Score

+ =
Weighted Weighted Composite Score
Section I, Part A Section I, Part B (Round to nearest
Score and Section II Score whole number)

AP Score Conversion Chart


United States History
Composite
Score Range AP Score
109-140 5
91-108 4
73-90 3
55-72 2
0-54 1

Downloaded by Loadster (kadenpizza66@gmail.com)


lOMoARcPSD|34158146

Question Descriptors and Performance Data

The following contains tables showing the content assessed,


the correct answer, and how AP students performed on each question.

Downloaded by Loadster (kadenpizza66@gmail.com)


lOMoARcPSD|34158146

2018 AP United States History


Question Descriptors and Performance Data

Multiple-Choice Questions

Question Learning Objective Key Concept Historical Thinking Skill Key % Correct
1 GEO-1.0 1.2 I Contextualization A 87
2 CUL-4.0 1.2 III Contextualization B 63
3 WOR-1.0 2.1 III Causation B 41
4 WOR-1.0 2.1 III Continuity and Change over Time C 81
5 WOR-1.0 2.1 I Comparison C 58
6 NAT-1.0 3.1 II Contextualization C 77
7 NAT-1.0 3.1 II Contextualization A 85
8 POL-2.0 4.1 III Causation D 46
9 CUL-3.0 4.1 III Comparison C 72
10 NAT-1.0 3.2 I Contextualization A 53
11 POL-3.0 3.2 III Contextualization D 72
12 WXT-2.0 4.1 I Analyzing Primary Sources A 67
13 NAT-2.0 5.2 I Continuity and Change over Time C 75
14 NAT-1.0 3.2 I Contextualization B 57
15 POL-2.0 4.1 III Contextualization C 87
16 CUL-3.0 5.3 II Continuity and Change over Time A 78
17 MIG-1.0 5.1 II Causation B 76
18 NAT-4.0 5.1 II Analyzing Secondary Sources A 44
19 CUL-4.0 7.2 I Comparison C 49
20 NAT-2.0 5.3 II Contextualization A 91
21 POL-2.0 5.2 II Causation C 90
22 NAT-1.0 5.3 I Analyzing Primary Sources D 57
23 WXT-3.0 4.2 I Causation B 66
24 WXT-2.0 6.1 II Causation B 61
25 WXT-2.0 4.2 III Continuity and Change over Time D 73
26 NAT-4.0 6.2 I Contextualization C 76
27 NAT-4.0 6.2 I Continuity and Change over Time A 56
28 POL-2.0 6.3 II Contextualization D 61
29 WOR-2.0 7.3 I Contextualization D 69
30 WOR-2.0 7.3 I Causation A 66
31 WOR-2.0 7.3 I Analyzing Primary Sources C 77
32 WOR-2.0 7.3 I Causation B 53
33 CUL-2.0 6.3 II Analyzing Primary Sources A 80
34 POL-2.0 8.2 III Comparison A 70
35 CUL-3.0 7.1 II Contextualization D 54
36 POL-3.0 7.1 III Contextualization B 38
37 WXT-1.0 6.1 II Continuity and Change over Time A 70
38 NAT-3.0 8.1 II Continuity and Change over Time C 25
39 WOR-2.0 7.3 III Analyzing Primary Sources C 60

Downloaded by Loadster (kadenpizza66@gmail.com)


lOMoARcPSD|34158146

2018 AP United States History


Question Descriptors and Performance Data

Question Learning Objective Key Concept Historical Thinking Skill Key % Correct
40 WXT-2.0 7.1 III Contextualization B 60
41 WOR-2.0 7.3 II Causation D 62
42 CUL-2.0 8.3 II Causation B 55
43 CUL-2.0 8.3 II Causation D 49
44 NAT-3.0 8.1 II Causation A 91
45 WXT-3.0 8.3 I Causation C 45
46 GEO-1.0 8.2 II Contextualization A 30
47 WXT-2.0 9.2 I Causation B 48
48 WXT-2.0 9.2 I Causation D 53
49 POL-2.0 8.2 II Contextualization C 82
50 WOR-2.0 8.1 II Analyzing Primary Sources C 57
51 POL-3.0 9.1 I Contextualization B 57
52 POL-3.0 8.2 III Contextualization A 77
53 POL-3.0 8.2 III Contextualization B 82
54 WXT-2.0 2.2 I Comparison A 70
55 WXT-1.0 5.2 I Causation B 52

Short Answer Questions

Historical Mean
Question Learning Objective Key Concept
Thinking Skill Score
Analyzing
1 NAT-2.0|POL-1.0 5.2 II 1.56
Secondary Sources
2 CUL-2.0|CUL-4.0 7.2 I Causation 1.63
3 CUL-1.0|CUL-2.0 2.2 I Comparison 1.35
4 WOR-1.0|NAT-3.0 8.1 I|8.1 II Comparison 2.11

Document-Based Question

Question Learning Objective Key Concept Historical Thinking Skill Mean Score
Analyzing Primary Sources|Contextualization|Continuity and Change over
1 WOR-2.0|POL-1.0 3.3 II|4.1 I 2.30
Time|Argument Development

Long Essay Questions

Mean
Question Learning Objective Key Concept Historical Thinking Skill
Score
2 NAT-1.0|CUL-2.0 2.2 I Contextualization|Causation|Argument Development 2.64
3 NAT-4.0|CUL-2.0|MIG-1.0 6.2 I|6.3 I Contextualization|Causation|Argument Development 2.25
4 CUL-2.0|WXT-3.0 8.3 I|8.3 II Contextualization|Causation|Argument Development 2.82

Downloaded by Loadster (kadenpizza66@gmail.com)


lOMoARcPSD|34158146

AP United States
History

The College Board


The College Board is a mission-driven not-for-profit organization that connects students to college
success and opportunity. Founded in 1900, the College Board was created to expand access to
higher education. Today, the membership association is made up of over 6,000 of the world’s leading
educational institutions and is dedicated to promoting excellence and equity in education. Each year,
the College Board helps more than seven million students prepare for a successful transition to college
through programs and services in college readiness and college success — including the SAT® and
the Advanced Placement Program®. The organization also serves the education community through
research and advocacy on behalf of students, educators, and schools. The College Board is committed
to the principles of excellence and equity, and that commitment is embodied in all of its programs, services,
activities, and concerns.

Downloaded by Loadster (kadenpizza66@gmail.com)

You might also like